免費論壇 繁體 | 簡體
Sclub交友聊天~加入聊天室當版主
分享
返回列表 发帖

一些代数和数论训练题

本帖最后由 hbghlyj 于 2021-4-2 12:51 编辑

标红的还未解决...
如果做错或繁了请直接指出
第一组
1.数列{$y_n$}满足$y_1$=1,n≥1时$13y_{n+1}-y_n^3-13y_n=0$,求证$y_n>\frac{17}2$
是否跟$e^{\frac{13}{2x^2}}$的牛顿法迭代有关
法①$y_{n+1}=y_n+\frac{y_n^3}{13}>\frac3{13}y_n-13,y_{20}>\frac13(\frac{16}{13})^{n-1}+\frac2{13},y_{20}\ge\frac13(\frac{16}{13})^{19}+\frac23>\frac13(1+19\cdot\frac{3}{13}+\frac{19\cdot18}2(\frac{3}{13})^2+\frac23>\frac{17}2$
法②$\because y_1=1,y_2=\frac{14}{13},\therefore y_3=\frac{14+\left(1+\frac1{13}\right)^3}{13}\ge\frac{14+1+C_3^1\times\frac1{13}+C_3^2\left(\frac1{13}\right)^2}{13}=\frac{15+\frac{42}{169}}{13}\ge 1.17$,同理$y_4>1.29,y_5>1.45,\because y_{n+1}-y_n=\frac{y_n^3}{13}>0,\therefore$数列$\{a_n\}$递增,$\because \frac{y_{n+1}}{y_n}=1+\frac{y_n^2}{13},\therefore \frac{y_{20}}{y_{19}}\ge\frac{y_{19}}{y_{18}}\ge\cdots\ge\frac{y_6}{y_5}=1+\frac{y_5^2}{13}\ge1.16,\therefore y_20>1.16^{15}\times1.45>(1+C_{15}^1\times0.16+C_{15}^2\times0.16^2)\times1.45>(3.4+100\times0.0256)\times1.45=8.642>\frac{17}2$
2.设$a_1,a_2,\cdots,a_n$为正数,求$S=\frac{(\sum\limits_{i=1}^n a_i^2)(\sum\limits_{i=1}^n \frac{a_i}{a_{i+1}+a_{i+2}+a_{i+3}})}{(\sum\limits_{i=1}^n a_i)^2}$的取值范围
令$a_2=a_3=\cdots=a_n=1$,S=$\frac{(a_1^2+n-1)(a_1+\frac{9}{a_1+2}+n-4)}{3(a_1+n-1)^2}$当n→∞时S→∞.
由柯西不等式$\sum\limits_{i=1}^n \frac{a_i}{a_{i+1}+a_{i+2}+a_{i+3}}\geq \sum\limits_{i=1}^n \frac{a_i^2}{a_i(a_{i+1}+a_{i+2}+a_{i+3})}\geq \frac{(\sum\limits_{i=1}^n a_i)^2}{\sum\limits_{i=1}^n a_i(a_{i+1}+a_{i+2}+a_{i+3})},S\geq\frac{\sum\limits_{i=1}^n a_i^2}{\sum\limits_{i=1}^n a_i(a_{i+1}+a_{i+2}+a_{i+3})}$,由排序不等式$S\geq\frac13$,当且仅当全相等时取等
3.已知$0<x_i,x_j<1(i,j\in\{0,1,2\cdots10\})$,证明:$\exists i,j\in\{0,1,2\cdots10\},0<x_ix_j(x_j-x_i)<\frac1{30}$
大概是用抽屉原理,具体不清楚
4.整数n≥3,对任意正数$a_i(i=1,2\cdots n)$,求$\sum\limits_{i=1}^n\frac{a_i}{a_{i+1}+a_{i+2}}$的取值范围
5.设$a_i\in[-2,2](i=1,2\cdots 2016),\sum\limits_{i=1}^{2016}a_i=0$,求$S=\sum\limits_{i=1}^{2016} a_i^3$的最大值
法①切线
法②x+y+z≤0时\(x^3+y^3+z^3≤3(\frac{x+y+z}3)^3\)
由于$S≤2016\cdot8$,S有最大值,取最大值时$a_i$可排列为$b_1=b_2=\cdots=b_k<0,2b_1+b_{k+1}>0$,否则可将不等的三个负数调成相等或把最小的正数变成负数使f不减而不变其和,这里调整法的说理有些问题。把一部分调成相等后另一部分就会变成不相等。即使配合反证法也不能自圆其说。求指教
法③令\(a_i=2\sin\alpha_i\),则\(S=-2\sum\limits_{i=1}^{2016}\sin3\alpha_i\leq4032\)
法④\(g(x)=x^3\)在[-2,0]上凸,在[0,2]下凸,设$a_1\cdots a_k\in[0,2],a_{k+1}\cdots a_{2016}\in[-2,0]$,对$f_1=\sum\limits_{i=1}ka_i^3$由Jensen不等式,当$a_1=\cdots=a_{[\frac t2]}=2,a_{[\frac t2]+1}=t-2[\frac t2],a_{[\frac t2]+2}=\cdots=a_k=0$时$f_1$最大
$f_2=\sum\limits_{i=[\frac t2]+1}^{2016}a_i^3\leq([\frac t2]-2016)\frac{t^3}{(2016-[\frac t2])^3},f=f_1+f_2\leq k\cdot2^3+(t-2[\frac t2])^3-\frac{t^3}{(2016-[\frac t2])^2}\leq8k-\frac{8k^3}{(2016-[\frac t2])^2}\leq4032$,当且仅当t=2k=1344时取等
6.n>1,求证$n^{(n+1)^{(n+2)^{n+3}}}>(n+3)^{(n+2)^{(n+1)^n}}$
$n^{(n+1)^{(n+2)^{n+3}}}=(n^2)^{\frac{(n+1)^{(n+2)^{n+3}}}2}\ge(n+3)^{\frac{(n+1)^{(n+2)^{n+3}}}2}\ge(n+3)^{\frac{(n+1)^{(n+2)^{n+3}}}{n+1}}=(n+3)^{(n+1)^{(n+2)^{n+3}-1}}>(n+3)^{(n+1)^{(n+2)^{n+2}}}=(n+3)^{((n+1)^2)^{\frac{(n+2)^{n+2}}2}}>(n+3)^{(n+2)^{(n+2)^{n+1}}}>(n+3)^{(n+2)^{(n+1)^{n}}}$
7.a,b,c>0,求证\(\sum\frac{ab}{a+b+2c}\leq\frac14(a+b+c)\)
不妨设p=a+b+c=3,q=ab+bc+ca,r=abc,则原不等式\( ⇔\sum\frac{ab}{3+c}\leq\frac34 ⇔162+63r\geq63q+4q^2\)(*)又\(27+9r=p^3+9r\geq4pq=12q\),所以\(r\geq\frac{4q-9}3\),欲证(*)只需证\(162+84q-189\geq63q+4q^2⇔(3-q)(-9+4q)\geq0\),而\(p^2\geq3q,3-q\geq0\),当且仅当a=b=c时取等.欲证(*)还得证\(-9+4q\le0\)的情形,此时\(162-63q-4q^2=(9-4q)(18+q)\ge0\)
8.多项式f(x)满足对任意实数x,\(e^x≥f(x)\)求f(x)的系数乘积最大值
9.求\(x^3 + 8 y^3 - x y=3\)的整数解
10.a,b,c,d≥0,a+b+c+d=4,求\(3(a^2+b^2+c^2+d^2)+4abcd\)的范围
最小值为16,当且仅当(1,1,1,1)或\(\frac43,\frac43,\frac43,0\)时取得
11.x,y,z>0,求\(\sum\frac{yz}{y^2+z^2+3x^2}\)的取值范围
最大值\(\frac35\)当且仅当x:y:z=1:1:1或3:3:2时取等
12.x,y,z≥0,x+y+z=4,求\(\sum \frac x{\sqrt{x+y}}\)的取值范围
最大值\(\frac52\)当且仅当0,3,1时取等
14.求有理数x,y,z,\(0≤x≤\frac12≤y≤z≤1,cos\pi x+\cos\pi y+\cos\pi z=0\),答案似乎是\(y=\frac12,z=1-x\vee y=\frac23-x,z=\frac23+x\)
15.a,b,c,d≥0,a+b+c+d=100,求$S=\sum\sqrt[3]{\frac a{b+7}}$的取值范围
引理.$\sqrt[3]{\frac x{t+7}}+\sqrt[3]{\frac t{x+7}}\leq\sqrt[3]{\frac{x+t+14}{7}}$①
对于x,y,z≥0,x+y≤z$\Leftrightarrow x^3+y^3+3xyz\leq z^3$,于是①$\Leftrightarrow\frac x{t+7}+\frac t{x+7}+3\sqrt[3]{\frac{xt(x+t+14)}{7(x+t)(t+7)}}\leq \frac{x+t+14}7$②
又由均值不等式,$3\sqrt[3]{\frac{xt(x+t+14)}{7(x+t)(t+7)}}=3\sqrt[3]{\frac{t(x+7)}{7(t+7)}\frac{x(t+7)}{7(x+7)}\frac{7(x+t+14)}{(x+t)(t+7)}}≤\frac{t(x+7)}{7(t+7)}+\frac{x(t+7)}{7(x+7)}+\frac{7(x+t+14)}{(x+t)(t+7)}$.从而,欲证②成立,只需证$\frac{t(x+7)}{7(t+7)}+\frac{t(x+7)}{7(t+7)}+\frac{x(t+7)}{7(x+7)}+\frac{7(x+t+14)}{(x+t)(t+7)}≤\frac{x+t+14}7$③.直接验证知③的等号成立.引理证毕.设x,y,z,t是变量的一个排列,x≤y≤z≤t,由排序不等式知S≤$(\sqrt[3]{\frac x{t+7}}+\sqrt[3]{\frac t{x+7}})+(\sqrt[3]{\frac y{z+7}}+\sqrt[3]{\frac z{y+7}})$,由引理得,S≤$\sqrt[3]{\frac{x+t+14}7}+\sqrt[3]{\frac{y+z+14}7}$,因为$f(u)=\sqrt[3]u$在$\mbb R^+$上是上凸函数,S≤2$\sqrt[3]{\frac{x+y+z+t+28}{14}}=\frac8{\sqrt[3]7}$.当且仅当(1,49,1,49)及其轮换时取等
16.a+b+c=3,求$3^a a+3^b b+3^c c$的取值范围
$\because y=3^x$单调递增,$\therefore (x-1)(3^x-3)\geq0,x3^x≥3x+3^x-3,3^a a+3^b b+3^c c≥3(a+b+c)+3^a+3^b+3^c-9=3^a+3^b+3^c≥3\cdot 3^{\frac{a+b+c}3}=9$
17.解关于x的不等式$2x^2-4x\sin x+1≤\cos2x$
$2x^2-4x\sin x+2\sin^2x≤0,x=\sin x,x=0(x>0,x>\sin x;x<0,x<\sin x)$
18.解关于x的不等式$(x+1)^3(x^3+5x)<10(x+1)^3+8$的解集
$f(x)=x^3+5x$严格递增,原不等式$\Leftrightarrow f(x)<f(\frac2{x+1})\Leftrightarrow x<\frac2{x+1},x\in(-∞,-2)\cup(-1,1)$
19.a,b,c>0,a+b+c=1,求u=$\sum\frac{3a^2-a}{1+a^2}$的取值范围
$f(x)=\frac{3x}{1+x^2}=\frac3{x+\frac1x}$在(0,1)严格递增,$\frac{3x^2-x}{1+x^2}=(f(x)-f(\frac13))(x-\frac13)\geq0,u\geq0$,当且仅当$a=b=c=\frac13$时取等
$u≤1,$当且仅当(1,0,0)时取等
20.$a_i>0(i=1,2\ldots n),\sum\limits_{1≤i<j≤n}\frac{a_ia_j}{a_i+a_j}\leq\frac n{2\sum\limits_{i=1}^na_i}\sum\limits_{1≤i<j≤n}a_ia_j$
记$S=\sum\limits_{i=1}^na_i$,利用恒等式$4a_ia_j=(a_i+a_j)^2-(a_i-a_j)^2,2n\sum\limits_{1≤i<j≤n}a_ia_j=(n-1)S^2-\sum\limits_{1≤i<j≤n}(a_i-a_j)^2$,原不等式$\Leftrightarrow(n-1)S-\sum\limits_{1≤i<j≤n}\frac{(a_i-a_j)^2}{a_i+a_j}\leq(n-1)S-\frac 1{S}\sum\limits_{1≤i<j≤n}(a_i-a_j)^2\Leftrightarrow S≥a_i+a_j$
21.$a_{n+1}=1-\arcsin\left(\frac π{a_n}\right),a_1=\frac π3$,求证:数列$\{a_n\}$收敛

7.a,b,c>0,求证\(\sum\frac{ab}{a+b+2c}\leq\frac14(a+b+c)\)
不妨设p=a+b+c=3,q=ab+bc+ca,r=abc,则原不等式\( ⇔\sum\frac{ab}{3+c}\leq\frac34 ⇔162+63r\geq63q+4q^2\)(*)又 ...
hbghlyj 发表于 2019-12-21 22:47
第7题其实用CS一步到位:`\displaystyle\sum\frac{ab}{a+b+2c}\leqslant\frac14\sum\left( \frac{ab}{a+c}+\frac{ab}{b+c} \right)=\frac14(a+b+c)`。

TOP

9.求\(x^3 + 8 y^3 - x y=3\)的整数解
hbghlyj 发表于 2019-12-21 22:47
9. 利用 `a^3+b^3+c^3-3abc` 的分解式,将方程变为 `x^3+(2y)^3+(1/6)^3-xy=3+(1/6)^3`,即可分解为
\[(6x+12y+1)(36x^2+144y^2+1-72xy-6x-12y)=649=11\times59,\]接下来大概就不难了。

TOP

回复 5# kuing
算了一下。第9题无整数解。

TOP

回复 7# hbghlyj


    是不是翻译成优超更合适?

TOP

回复 8# facebooker

随便吧……
其实不如直接用符号吧…… `\succ` (\succ) and `\prec` (\prec)

TOP

本帖最后由 hbghlyj 于 2020-4-18 21:54 编辑

1.求最小常数C,使得对任意正数$a_i(i=1,2\ldots2019)$,均有$\sum\limits_{i=1}^{2019}\frac{a_{i+3}-a_{i+1}}{a_{i+3}+a_{i}}≤C$
当$a_{i+3}$远大于$a_i$时,原式$\to0$,构造$1,q,q^2\ldots q^{2015},q^{2016},q^{2016},q^{2016}$,此时$\sum\limits_{i=1}^{2019}\frac{a_{i+3}-a_{i+1}}{a_{i+3}+a_{i}}=2014\frac{1+q}{1+q^3}+\frac{2q^{2016}}{q^{2016}+1}+\frac{2q^{2016}}{q^{2016}+q}+\frac{q^{2016}+1}{q^{2016}+q^2}+\frac{q^{2016}+q^{2015}}{q^{2016}+q^{2013}}$当q$\to\infty$时$\sum\limits_{i=1}^{2019}\frac{a_{i+3}-a_{i+1}}{a_{i+3}+a_{i}}\to6$
记$S_1=\sum\limits_{i≡1\pmod3}a_i,S_2=\sum\limits_{i≡2\pmod3}a_i,S_3=\sum\limits_{i≡0\pmod3}a_i$,则$\sum\limits_{i=1}^{2019}\frac{a_{i+1}+a_{i}}{a_{i+3}+a_{i}}>\sum\limits_{i=1}^{2019}\frac{a_{i+1}+a_{i}}{S_i}=\sum\limits_{i=1}^{2019}\frac{a_i}{S_i}+\sum\limits_{i=1}^{2019}\frac{a_{i+1}}{S_i}=\frac{S_1}{S_1}+\frac{S_2}{S_2}+\frac{S_3}{S_3}+\frac{S_2}{S_1}+\frac{S_3}{S_2}+\frac{S_1}{S_3}≥6,\sum\limits_{i=1}^{2019}\frac{a_{i+3}-a_{i+1}}{a_{i+3}+a_{i}}=2019-\sum\limits_{i=1}^{2019}\frac{a_{i+1}+a_{i}}{a_{i+3}+a_{i}}≤2019-6=2013$
2.
求最小的常数$\lambda=\lambda(n,k),n,k\in\mbb N,n≥4k$,使得对任意$a_1,a_2\ldots a_n>0,$均有$\sum\limits_{i=1}^n\frac{a_i}{\sqrt{a_i^2+\ldots+a_{i+k}^2}}≤\lambda$
3.求最大的常数$k_n$使得对任意正数$a_1,a_2\cdots a_n$均有$(\sum\limits_{i=1}^na_i^2)(\sum\limits_{i=1}^n\frac{a_i}{a_{i-1}+a_{i+1}})≥k_n\sum\limits_{i=1}^n(a_i-a_{i+1})^2$
取n=2,$k\leq\frac{(a^2+b^2)(\frac ba+\frac ab)}(x:=\frac ab)=\frac 14(1+\frac2{x+\frac1x-2})(x+\frac1x)$当$x+\frac1x=4$即$\frac ab=2\pm\sqrt3$时取最小值2,故$k_2\leq2$
$n\geq3$时$\frac{(\sum\limits_{i=1}^na_i^2)(\sum\limits_{i=1}^n\frac{a_i}{a_{i-1}+a_{i+1}})}{\sum\limits_{i=1}^n(a_i-a_{i+1})^2}=\frac{(\sum\limits_{i=1}^na_i^2)(\sum\limits_{i=1}^n\frac{a_i^2}{a_ia_{i-1}+a_ia_{i+1}})}{2(\sum\limits_{i=1}^na_i^2-\sum\limits_{i=1}^na_i^2a_ia_{i+1})}\geq\frac{(\sum\limits_{i=1}^na_i^2)(\sum\limits_{i=1}^n\frac{a_i^2}{\sum\limits_{i=1}^na_i^2a_ia_{i+1}})}{2(\sum\limits_{i=1}^na_i^2-\sum\limits_{i=1}^na_i^2a_ia_{i+1})}$,令$\sum\limits_{i=1}^na_i^2=a,\sum\limits_{i=1}^na_ia_{i+1}=b,$化为$\frac {a^2}{2b(a-b)}\geq \frac {a^2}{2(\frac a2)^2}=2$
4.
给定正整数n,求$c_n,d_n$的最大值,使得对任意$\prod\limits_{i=1}^na_i=1$,均有$\sum\limits_{i=1}^n\frac1{a_1+1}≥c_n,\sum\limits_{i=1}^n\frac1{a_1+2}≥d_n$
5.给定正整数n,求最大的正整数$d_n$,使得对任意$\sum\limits_{i=1}^na_i=n(a_i>0)$,均有$\sum\limits_{i=1}^n\frac1{d_n+a_i^2}≥\frac n{d_n+1}$
6.$x_1x_2\cdots x_n=1(x_i>0)$,证明$n^n\prod\limits_{i=1}^n(x_i^n+1)≥(\sum\limits_{i=1}^nx_i+\sum\limits_{i=1}^n\frac1{x_i})^n$
法①只需证局部不等式$\sqrt[n]{\prod\limits_{i=1}^n(1+x_i^n)}≥x_n+\frac1{x_n}$
由均值不等式$\frac{x_1^n}{1+x_1^n}+\frac{x_2^n}{1+x_2^n}+\cdots+\frac{x_{n-1}^n}{1+x_{n-1}^n}+\frac1{1+x_n^n}≥\frac n{x_n\sqrt[n]{\prod\limits_{i=1}^n(1+x_i^n)}},\frac1{1+x_1^n}+\frac1{1+x_2^n}+\cdots+\frac1{1+x_{n-1}^n}+\frac{x_n^n}{1+x_n^n}≥\frac {nx_n}{\sqrt[n]{\prod\limits_{i=1}^n(1+x_i^n)}}$,相加即得.
法②$LHS=\sqrt[n]{\prod\limits_{i=1}^nn(1+x_i^n)}=\sqrt[n]{\prod\limits_{i=1}^n(\underbrace{1+\ldots+1}_{n个}+\underbrace{x_i^n+\ldots+x_i^n}_{n个})}$
由Hölder不等式,
$\prod\limits_{i=1}^n(1+\underbrace{x_1^n+\ldots+x_1^n}_{n-1个}+x_1^n+\underbrace{1+\ldots+1}_{n-1个})(x_2^n+1+\underbrace{x_2^n+\ldots+x_2^n}_{n-2个}+1+x_2^n+\underbrace{1+\ldots+1}_{n-2个})(x_3^n+x_3^n+1+\underbrace{x_3^n+\ldots+x_3^n}_{n-3个}+1+1+x_3^n+\underbrace{1+\ldots+1}_{n-3个})\ldots(\underbrace{x_n^n+\ldots+x_n^n}_{n-1个}+\underbrace{1+\ldots+1}_{n个}+x_n^n)≥(
x_2x_3\
cdots x_n+x_1x_3\cdots x_n+x_1x_2\ldots x_{n-1}+x_1+x_2+\cdots+x_n)^n=(\sum\limits_{i=1}^n\frac1{x_i}+\sum\limits_{i=1}^nx_i)$7.$a_0+a_1+\cdots+a_k=S_k(a_i>0)$,证明$\sqrt[n]{\prod\limits_{i=1}^n\frac{a_i}{S_i}}+\sqrt[n]{\frac{a_n}{S_n}}≤1$
8.
$a_i>0(i=1,2\ldots n),g_n=\sqrt[n]{a_1\cdots a_n},A_n=\frac{a_1\cdots a_n}n,G_n=\sqrt[n]{A_1\cdots A_n}$,证明$n\sqrt[n]{\frac{G_n}{A_n}+\frac{g_n}{G_n}}≤n+1$
9.
$a_i>0(i=1,2\ldots n),$证明$\left(\frac{\sum\limits_{i=1}^n\sqrt[j]{a_1\cdots a_j}}{\sum\limits_{i=1}^n a_j}\right)^{\frac1n}+\frac{\sqrt[n]{a_1\cdots a_n}}{\sum\limits_{j=1}^n \sqrt[j]{a_1\cdots a_n}}≤\frac{n+1}n$
10.$0≤a_1≤a_2≤\cdots≤a_n,a_1≤\frac12a_2≤\cdots≤\frac1na_n,G_n=\sqrt[n]{a_1\cdots a_n},A_n=\frac{a_1\cdots a_n}n,$证明$\frac{A_n}{G_n}≤\frac{n+1}{2\sqrt[n]{n!}}$
11.$\sum\limits_{i=1}^n\frac1{x_i+2019}=\frac1{2019}(x_i\in\mathbb R_+)$,证明$\frac{\sqrt[n]{x_1x_2\ldots x_n}}{n-1}≥2019$
作代换$x_i=2019\frac{a_1+a_2+\cdots+a_{i-1}+a_{i+1}+\cdots+a_n}{a_i}$,$a_i>0$,则$\frac{\sqrt[n]{x_1x_2\ldots x_n}}{n-1}=2019\frac{\sqrt[n]{\prod\limits_{i=1}^n\frac{a_1+a_2+\cdots+a_{i-1}+a_{i+1}+\cdots+a_n}{a_i}}}{n-1}≥2019\frac{\sqrt[n]{\prod\limits_{i=1}^n\frac{(n-1)\sqrt[n-1]{a_1a_2\cdots a_{i-1}a_{i+1}\cdots a_n}}{a_i}}}{n-1}=2019$
12.求所有实数C,使得存在函数f:$\mathbb R_{≥0}\to\mathbb R$,对任意x,y≥0均有$f(x^2+y)≥cf(y)+x$
13.a>c>b>d>0,求$S=\sum\frac a{a+b}$的取值范围
$\frac a{a+b}+\frac b{b+c}=1+\frac b{b+c}-\frac b{b+a}=1+\frac{b(a-c)}{(b+c)(b+a)}$,$f(x)=\frac {x(a-c)}{(x+c)(x+a)}=\frac {a-c}{x+\frac{ac}x+a+c}$在(0,c)单调递增,f(b)>f(d),S=2+f(b)-f(d)>2(或者$S=\sum\frac1{1+\frac bd}≥\frac{4^2}{4+\sum\frac ba}>\frac{4^2}{4+1+1+1+1}=2$),$S=1+\frac d{d+a}-\frac b{a+b}+\frac b{b+c}+\frac c{c+d}<1+0+\frac12+1=\frac52$,当且仅当a→∞,b=c,d=0时取等.
14.求方程组$5(x+\frac1x)=12(y+\frac1y)=13(z+\frac1z)$,xy+yz+zx=1的所有实数解(x,y,z)
易知x,y,z同号,当x,y,z>0时,$x=c\tan\frac A2...(A,B,C\in(0,\pi))$,则$A+B+C=\pi,\sin A:\sin B:\sin C=5:12:13,C=\frac \pi2,z=\tan\frac C2=1,(x,y,z)=(\frac15,\frac23,1)$,当x,y,z>0时,$(x,y,z)=(-\frac15,-\frac23,-1)$
15.$5(x+\frac1x)=12(y+\frac1y)=13(z+\frac1z)$,求xy+yz+zx的取值范围
min{x y + y z + z x|5 (x + 1/x) = 12 (y + 1/y) = 13 (z + 1/z)}≈1.51191 at (x, y, z)≈(0.0799469, 5.04696, 0.216196)
min{x y + y z + z x|5 (x + 1/x) = 12 (y + 1/y) = 13 (z + 1/z)}≈13.4912 at (x, y, z)≈(5.06983, 1.54907, 0.851761)
16.$x_i≥0,\sum\limits_{i=1}^nx_i=4,$求证$\sum\limits_{cyc}x_1^3x_2≤27$
先证三元情形,不妨设$a=\max\{a,b,c\}$,当b<c时由于$\sum\limits_{cyc}a^3b-\sum\limits_{cyc}ab^3=(a-b)(b-c)(a-c)(a+b+c)<0$,将b,c交换后原式的值增大,下证b≥c的情形
$a^3b+b^3c+c^3a≤a^3b+2a^2bc≤a^3b+3a^2bc=a^2b(a+3c)=\frac13a\cdot a\cdot3b\cdot(a+3c)≤\frac13(\frac34(a+b+c))^4=27$,当且仅当3,1,0时取等
后同18#,n元情形当且仅当3,1,其余都是0时取等
当且仅当
推广:
$x,y,z\ge0,x+y+z=1,n,m\in\mathbf N_+,n\ge m,$则$x^ny^m+y^nz^m+z^nx^m\le\led \frac13,n=1\\\frac{n^nm^m}{(n+m)^{n+m}},n>1\endled$
n>1时等号成立当且仅当x,y,z为$\frac n{n+m},\frac m{n+m},0$及其轮换
不妨设$x\ge y,x\ge z$,
1°n>1=m,这时有$x^ny+y^nz+z^nx\le x^{n-1}(xy+yz+z^2)\le x^{n-1}(y(x+z)+\frac12z(x+z))=\frac 1nx^{n-1}(x+z)(ny+\frac n2z)\le \frac1n\left[\frac{(n-1)x+(x+z)+(ny+\frac n2z)}{n+1}\right]^{n+1}=\frac1n\left[\frac{n(x+y)+\frac{n+2}2z}{n+1}\right]^{n+1}\le\frac1n\left[\frac{n(x+y+z)}{n+1}\right]^{n+1}=\frac{n^n}{(n+1)^{n+1}}$
2°n≥m>1,这时有
$x^ny^m+y^nz^m+z^nx^m\le x^n(y^m+y^{m-1}z+z^m)\le x^n(y+z)^m=\frac1{n^mm^n}(mx)^n(ny+nz)^m\le\frac1{n^mm^n}\left[\frac{n(mx)+m(ny+nz)}{n+m}\right]^{n+m}=\frac1{n^mm^n}\left[\frac{mn(x+y+z)}{n+m}\right]^{n+m}=\frac{n^nm^m}{(n+m)^{n+m}}$
综上两种情形知不等式成立,且等号成立的条件都是
$\led z=0,\\mx=ny+nz,\\x+y+z=1,\endled$即$x=\frac n{n+m},y=\frac m{n+m},z=0$
若$x_1,x_2,\cdots,x_p(p\ge3)$是满足$x_1+x_2+\cdots+x_p=1$的非负实数,$n,m\in\mathbf N_+,n\ge m,n>1$则
$x_1^nx_2^m+x_2^nx_3^m+\cdots+x_{p-1}^nx_p^m+x_p^nx_1^n\le \frac{n^nm^m}{(n+m)^{n+m}}$,等号成立当且仅当$x_i=\frac n{n+m},x_{i+1}=\frac m{n+m}(i=1,2,\cdots,p;x_{p+1}=x_1)$,其余各项均为0.
证明:不妨设$x_1\ge x_i(i=2,3,\cdots,p)$,
1°当p=3时,由推广1可知不等式成立
2°假设当p=k时不等式成立,则$x_1^nx_2^m+x_2^nx_3^m+\cdots+x_k^nx_{k+1}^m+x_{k+1}^nx_1^m\le x_1^nx_2^m+x_1^nx_3^m+(x_2+x_3)^nx_4^m+\cdots+x_k^nx_{k+1}^m+x_{k+1}^nx_1^m\le x_1^n(x_2+x_3)^m+(x_2+x_3)^nx_4^m+\cdots+x_k^nx_{k+1}^m+x_{k+1}^nx_1^m\le \frac {n^nm^m}{(n+m)^{n+m}}$这表明,当p=k+1时不等式也成立,故对任何$p\ge3$不等式都成立.
定理3.若$x_1,x_2,\cdots,x_p(p\ge4)$是满足$x_1+x_2+\cdots+x_p=1$的非负实数,则$x_1x_2+x_2x_3+\cdots+x_{p-1}x_p+x_px_1\le \frac14$
其中p=4时的证明是$x_1x_2+x_2x_3+x_3x_4+x_4x_1=(x_1+x_3)(x_2+x_4)\le\left(\frac{x_1+x_2+x_3+x_4}2\right)^2=\frac14$当且仅当$x_1+x_3=x_2+x_4=\frac12$时取等号.
若$x_1,x_2,\cdots,x_p(p\ge3)$是满足$x_1+x_2+\cdots+x_p=q(q>0)$的非负实数,$n,m\in\mathbf N_+$,则$x_1^nx_2^m+x_2^nx_3^m+\cdots+x_{p-1}^nx_p^m+x_p^nx_1^m\le\led n^nm^m\left(\frac q{n+m}\right)^{n+m},n,m不全为1或p\ge4\\\frac{q^2}3,n=m=1且p=3\endled$
17.给定正整数n,对任意正数a,b使得$\frac1a+\frac1b=1,$求$(a+b)^n-a^n-b^n$的取值范围
a=b=2时取得最小值.下面证$(a+b)^n-a^n-b^n≤2^{2n}-2^{n+1}$
当n=1时显然成立.设n=k时命题成立.由$\frac1a+\frac1b=1,$有ab=a+b≥4,则 $(a+b)^{k+1} =(a+b)^k(a+b)≥(a+b)(a^k +b^k )+(2^{2k} −2^{k+1})(a+b)=a^{k+1}+b^{k+1}+ba^k +ab^k +(2^{2k} −2^{k+1})(a+b)=a^{k+1}+b^{k+1}+ab(a^{k-1}+b^{k-1}+2^{2k} −2^{k+1}) ≥a^{k+1}+b^{k+1}+2^{2(k+1) }−2^{k+2}.$
即当 n = k + 1 时,不等式也成立. 综上,由归纳法原理,对任意 $n \in\mathbb N $不等式成立.
最后一个≥用了均值不等式$\sqrt[k-1]{\frac{a^{k-1}+b^{k-1}}2}≥\frac{a+b}2≥2,a^{k-1}+b^{k-1}≥2^{k}$.
另一种形式:
$(a+b)^{k+1}≥a^{k+1}+a^k+\cdots+a^2+a+b^{k+1}+b^k+\cdots+b^2+b+ab(1+2^{2k} −2^{k+1})$
18.给定正整数r,对任意$a_1,a_2\cdots a_n$满足$\sum\limits_{i=1}^na_i=n(n-r),\sum\limits_{i=1}^na_i^2=n^2(n-r)$,求$2n\sum\limits_{i=1}^na_i^3-\sum\limits_{i=1}^na_i^4$的范围
$2n\sum\limits_{i=1}^na_i^3-\sum\limits_{i=1}^na_i^4≤n^2\sum\limits_{i=1}^na_i^2=n^4(n-r)$,取等当且仅当$a_1,a_2\cdots a_n$有r个0,n-r个n
最小值猜测是r个2n-2r,n-r个n-2r时取得.
得到了最值后,是否能取得它们之间的每个实数?这也是个问题

TOP

本帖最后由 hbghlyj 于 2019-12-29 10:31 编辑

第三组.一般连分式理论、线性分式递推数列
1.在连分式$\frac{b_1}{a_1-}\frac{b_2}{a_2-}\frac{b_3}{a_3-}\cdots$中,如果$a_i≥b_i+1>0$,则该连分式的渐进分数是递增的一列正分数
证明:根据假设$\frac{b_1}{a_1},\frac{b_2}{a_2},\frac{b_3}{a_3}\cdots$为真分数,这些分数
的分母至少比分子大1。二阶渐近分数为$\frac{b_1}{a_1-\frac{b_2}{a_2}}<\frac{b_1}{a_1-1}≤1$为一真分数。同理,$\frac{b_2}{a_2-\frac{b_3}{a_3}}$是真分数,记为$f_1$,这样三阶渐近分数为$\frac{b_1}{a_1-f_1}<\frac{b_1}{a_1-1}≤1$,所以也是真分数.以此类推。
又因为$p_n=ap_{n-1}-bp_{n-2},q_n=aq_{n-1}-bq_{n-2}$,所以$\frac{p_{n+1}}{q_{n+1}}-\frac{p_{n}}{q_{n}}=\frac{b_{n+1}q_{n-1}}{q_{n+1}}(\frac{p_{n}}{q_{n}}-\frac{p_{n-1}}{q_{n-1}})$,$\frac{p_{n}}{q_{n}}-\frac{p_{n-1}}{q_{n-1}}$与$\frac{p_{n+1}}{q_{n+1}}-\frac{p_{n}}{q_{n}}$同号。又$\frac{p_2}{q_2}-\frac{p_1}{q_1}=\frac{a_2b_1}{a_1a_2-b_2}-\frac{b_1}{a_1}=\frac{b_1b_2}{q_1q_2}>0$,所以为正,$0<\frac{p_1}{q_1}<\frac{p_2}{q_2}<\frac{p_3}{q_3}<\ldots$。
2.数列{$a_n$}满足$a_{n+1}=\frac1{2-a_n},\forall_{n\in\mbb N_+}a_{n+1}>a_n,$求$a_1$的取值范围
若$a_1>2$,则$a_2<0,$故$a_1<2,$由1,$a_1$的取值范围是(-∞,1)作图
3.$f(x)=\frac x{1+}\frac x{1+}\ldots$,求f'(x)

TOP

本帖最后由 hbghlyj 于 2020-4-22 14:45 编辑

第四组
1.连续函数f:$\mathbb R\to\mathbb R$满足$f(f(f(f(x))))=kx^{81}\forall x\in\mathbb R$,求证k≥0
如果k=0,则结论已经成立.下设k≠0.若f(a)=f(b),则$ka^{81}=f(f(f(f(a))))=f(f(f(f(b))))=kb^{81},a=b$,f是单射,又因为f是连续函数,所以f单调.两边取f值,得到$f(kx^{81})=kf^{81}(x)$,
如果f(0)≠0,有$f(0)=kf^{81}(0)$,k>0.
如果f(0)=0,当f(x)是递增函数时x,对x>0,f(f(f(f(x))))>0,所以$kx^{81}$>0,k>0.当f(x)是递减函数时,对x>0,f(x)<0,f(f(x))>0,f(f(f(x)))<0,f(f(f(f(x))))>0,所以$kx^{81}$>0,k>0.
2.求所有正实数s和t,使得对任意正整数n,均有$\lfloor tn\rfloor-\lfloor \lfloor sn\rfloor\rfloor=1$
3.min{ab,bc,ca}≥1,证明$\sqrt[3]{\prod(a^2+1)}≤(\frac{\sum a}3)^3+1$
4.非零实数$a_i(i=1,2\ldots8)$,求证:六数$a_1a_3+a_2a_4,a_1a_5+a_2a_6,a_1a_7+a_2a_8,a_3a_5+a_4a_6,a_3a_7+a_4a_8,a_5a_7+a_6a_8$中至少有一个非负数
5.用f(x)表示x的最大因数$(x,n∈\mathbf N_+)$,求证:$\frac{2n}3<\sum\limits_{x=1}^n\frac{f(x)}x<\frac{2(n+1)}3$
6.a,b,c≥0,求证$\sum\sqrt{\frac{7a}{a+3b+3c}}≤3$
由柯西不等式,$\sum\sqrt{\frac{7a}{a+3b+3c}}≤(\sum(17a+2b+2c))^{\frac12}\left(\sum\frac{7a}{(17a+2b+2c)(a+3b+3c)}\right)^{\frac12}$
只需证$(a+b+c)\sum\frac{49a}{(17a+2b+2c)(a+3b+3c)}≤3\Leftrightarrow\sum\left(1-\frac{49a(a+b+c)}{(17a+2b+2c)(a+3b+3c)}\right)≥0\Leftrightarrow\sum\frac{(b+c-2a)(8a+3b+3c)}{(17a+2b+2c)(a+3b+3c)}≥0$(不妨设a+b+c=1)$\Leftrightarrow\sum\frac{(b+c-2a)(5a+3)}{(15a+2)(3-2a)}≥0\Leftrightarrow\sum(a-b)()$
7.给定正整数1<k≤n,对于实数$a_1,a_2\cdots a_n,\sum\limits_{i=1}^na_i^2+\sum\limits_{i=1}^{n-1}a_ia_{i+1}=1,$求$\max\{\abs {a_k}\}$
配方得$a_1^2+(a_1+a_2)^2+\cdots+(a_{n-1}+a_n)^2+a_n^2=2$,由均值不等式,三角不等式有$\sqrt{\frac{a_1^2+(a_1+a_2)^2+\cdots+(a_{k-1}+a_k)^2}{k}}≥\frac{\abs{a_1}+\abs{-a_1-a_2}+\cdots+\abs{(-1)^{k-1}a_{k-1}+(-1)^{k-1}a_k}}k≥\frac{\abs{a_k}}k,a_1^2+(a_1+a_2)^2+\cdots+(a_{k-1}+a_k)^2≥\frac{a_k^2}k$,同理$(a_k+a_{k+1})^2+\cdots+(a_{n-1}+a_n)^2+a_n^2≥\frac{a_k^2}{n-k+1}$,$\frac{a_k^2}k+\frac{a_k^2}{n-k+1}≤2$,当且仅当$a_i= \begin{cases}
    (-1)^{k+i}\frac{ia_k}k  &1≤i ≤k-1\\
\pm\sqrt{\frac{2k(n-k+1)}{n-1}},&i=k\\
    (-1)^{i-k}\frac{(n-i+1)a_k}{n-k+1},& k+1≤i≤n
\end{cases} $时取等
8.西尔维斯特数列(Sylvester's sequence)$a_n=a_0a_1...a_{n-1}+ 1, a_0 = 2$.A00058
$a_{n+1}=a_0a_1...a_{n}+ 1= (a_n-1)a_n+1=a_n^2 - a_n+ 1$,所以它是一个二次函数的迭代
2, 3, 7, 43, 1807, 3263443...
显然两个相异的西尔维斯特数必定互质。前四项都是素数,大部分是合数比如1807=13×139. 10650056950807 = 547 ×19569939581.在首三百万个质数只有1166个是西尔维斯特数。现时所知的西尔维斯特数中,都是无平方数因数的数,但未有证明所有西尔维斯特数都是。
推广:对于任意与正整数m互素的$a_0$,序列$a_{n+1}=a_n^2 - ma_n+ m$中每两项是互质的。
证明:
自然数x_i两两互质,$f(1)=x_1,f(2)=x_2\ldots f(k)=x_k,\forall n>k:f(n)=f(1)\ldots f(n-1)+1$
一个有趣性质是n≥2,$\frac1{a_0}+\frac1{a_1}+\ldots+\frac1{a_{n-1}}=1-\frac1{a_n-1}$。因此,我们还可以写出$a_n=\frac{\frac1{a_0}+\frac1{a_1}+\ldots+\frac1{a_{n-1}}-2}{\frac1{a_0}+\frac1{a_1}+\ldots+\frac1{a_{n-1}}-1}$
这个性质给出无穷多种1的埃及分数表示1 = 1/2 + 1/3 + 1/6 = 1/2 + 1/3 + 1/7 + 1/42 = 1/2 + 1/3 + 1/7 + 1/43 + 1/1806 = ...
1的贪婪算法埃及分数表示为$ 1 = \frac12 + \frac13 + \frac17 + \frac1{43} + \frac1{1807} +...$
下面的一个性质说明它在埃及分数序列中对1的逼近是最优的.
正整数列$\{b_n\}$满足$\sum\limits_{i=0}^n\frac1{b_n}<1$,则$\sum\limits_{i=0}^n\frac1{b_n}≤\sum\limits_{i=0}^n\frac1{a_n}$
证明:用反证法和第二数学归纳法。假设结论对$1,2\cdots n-1$成立,对n不成立.由于$\sum\limits_{i=0}^n\frac1{b_i}$的分母为$\prod\limits_{i=0}^nb_i$的正因数,$1-\frac1{\prod\limits_{i=0}^nb_i}≥\sum\limits_{i=0}^n\frac1{b_i}>\sum\limits_{i=0}^n\frac1{a_i}=1-\frac1{\prod\limits_{i=0}^na_i},\prod\limits_{i=0}^nb_i>\prod\limits_{i=0}^na_i$(*).不妨设$b_i≤b_{i+1}$,用Abel变换,$\sum\limits_{i=0}^n\frac{b_i}{a_i}=\sum\limits_{i=0}^n(b_i-b_{i+1})\sum\limits_{j=0}^{i}\frac1{a_j}+b_n\sum\limits_{i=0}^{n}\frac1{a_i}<\sum\limits_{i=0}^n(b_i-b_{i+1})\sum\limits_{j=0}^{i}\frac1{b_j}+b_n\sum\limits_{i=0}^{n}\frac1{b_i}=\sum\limits_{i=0}^n\frac{b_i}{b_i}=n+1$,由均值不等式$\prod\limits_{i=0}^n\frac{b_i}{a_i}<1$,与(*)矛盾。
对于$\frac1{x_1}+\frac1{x_2}+\ldots+\frac1{x_n}+\frac1{x_1x_2\ldots x_n}=1$型不定方程,$x_1,x_2,\ldots ,x_n\in \mathbf N_+$,前n个西尔维斯特数是其一组解,n=4,5时方程只有一组解,n=6时

$a_k^2 + 1|a_{k+1}^2 + 1$
证明:$a_{k+1}^2 + 1 = (a_k^2 - a_k+ 1)^2 +1 = (a_k^2 -2a_k+2)(a_k^2 + 1) $
$a_{k+1}+a_k|a_ka_{k+1}+ 1$
证明:$a_ka_{k+1}-1=a_k(a_k^2-a_k+1)-1=a_k^3-a_k^2+a_k-1=(a_k^2+1)(a_k-1)=(a_{k+1}+a_k)(a_k-1)$

考虑映射$f(a/b)=\frac{a^3+b}{a+b^3}$。从a=1,b=2开始,在每个新的既约分数上反复进行这个映射,得到1/2,1/3,1/7,1/43,1/1807...
证明:

$a_n=\lfloor{c^{2^(n+1)} + \frac12}\rfloor$,其中Vardi常数c= 1.2640847353053011130795995...

-3是西尔维斯特数的任何素因子的二次剩余。
证明:

如果取包含前n项的倒数的序列的任何子集,且首项变号,则该子集的元素和等于元素积。因此-1/2=-1/2、-1/2+1/3=-1/2*1/3、-1/2+1/3+1/7=-1/2*1/3*1/7、-1/2+1/7+1/43=-1/2*1/3*1/7*1/43等等。
证明:
此序列还与勾股数组的短边有关,(3,4,5),(5,12,13),(13,84,85),.A053630(N)=2*a(N)-1。
证明:
9.设$x_1,x_2\cdots x_n$为正实数,$x_1x_2\cdots x_n=1$,证明$(\sqrt2-x_1)(\sqrt2-x_2)\ldots(\sqrt2-x_n)≥(\sqrt2+1)^n$
此为Hölder不等式的特例
法①(反向归纳法)$(\sqrt2-a)(\sqrt2-b)≥(\sqrt{2}-\sqrt{ab})^2\Leftrightarrow(\sqrt a-\sqrt b)^2≥0$,命题对2的幂成立,设n≤$2^k$,取$x_1x_2\cdots x_n,\underbrace{1,1\cdots1}_{2^k-n个}$,命题对一切正整数n成立
法②假设n=k时命题成立,当n=k+1时,不妨设$x_1≤x_2≤\cdots≤x_i≤ x_{i+1}≤\cdots≤x_k≤x_{k+1}$,此时有$(x_i-1)(x_{i+1}-1)≤0$,由此可得$(\sqrt2+x_i)(\sqrt2+x_{i+1})≥(\sqrt2+1)(\sqrt2+x_ix_{i+1})$,由归纳假设$(\sqrt2-x_1)(\sqrt2-x_2)\ldots(\sqrt2-x_n)≥(\sqrt2-x_1x2)(\sqrt2-x_3)\ldots(\sqrt2-x_n)≥(\sqrt2+1)(\sqrt2+1)^n$
法③由均值不等式$1=\frac1n\sum_{i=1}^n\frac{\sqrt2+x_i}{\sqrt2+x_i}=\frac1n\sum_{i=1}^n\frac{\sqrt2}{\sqrt2+x_i}+\frac1n\sum_{i=1}^n\frac{x_i}{\sqrt2+x_i}≥\sqrt[n]{\prod\limits_{i=1}^n\frac{\sqrt2}{\sqrt2+x_i}}+\sqrt[n]{\prod\limits_{i=1}^n\frac{x_i}{\sqrt2+x_i}}=\frac{\sqrt2+1}{\prod\limits_{i=1}^n(\sqrt2+x_i)}$
法④(琴生不等式)令$x_i=e_{t_i}$,则$\sum\limits_{i=1}^nt_i=0$,设$f(t)=ln(\sqrt2+e^t)$.由于$f''(t)=\frac{\sqrt2e^t}{(\sqrt2+e^t)^2>0}$,故f(x)为下凸函数.所以$\frac1n\sum\limits_{i=1}^nln(\sqrt2+e^{t_i})≥ln(\sqrt2+e^{\frac1n\sum\limits_{i=1}^nt_i})=ln(\sqrt2+1)$,从而$(\sqrt2+x_1)(\sqrt2+x_2)\cdots(\sqrt2+x_n)≥(\sqrt2+1)^n$
10.x,y,z>0,$x^2+y^2+z^2=3$,求证$\sum\frac1{4-x}≤1$
待定常数$\alpha$,使$\frac1{4-x}≤\alpha(x^2-1)+\frac13$,令$f(x)=\frac1{4-x}-\alpha(x^2-1)-\frac13$则f'(x)=$\frac1{(4-x)^2}-2\alpha x$,由f'(1)=0解得$\alpha=\frac1{18}$.下面验证其正确性
$\frac1{4-x}-\frac1{18}$
11.求常数k的范围,使得对任意非负数a,b,c满足$a^2+k,b^2+k,c^2+k$≥0,abc=1,都有
$\sum\sqrt{a^2+k}≤\sqrt{1+k}(a+b+c)$
待定常数$\alpha,f(x)=\sqrt{x^2+k}-\sqrt{1+k}x+\alpha ln x$,即证$\sum f(a)≥0$.$f'(x)=\frac{x }{\sqrt{k + x^2}}- \sqrt{k + 1} +\frac{\alpha}x$,注意到f(1)=0,由f'(1)=0解得$\alpha=\frac k{\sqrt{k + 1}}$.$f''(x)=\frac k{(k + x^2)\sqrt{k + x^2}}-\frac{\alpha}{x^2}$.只需求这样的k使f''(1)<0,$f''(1)=\frac k{(k + 1)\sqrt{k + 1}}<\frac k{\sqrt{k + 1}},k\in(-1,0)\cup(0,+\infty)$,所以$k\in[-1,+\infty)$,其中k=0时是恒等式,k=1时只有一组(1,1,1)使根式有意义,其他情况下当且仅当(1,1,1)时取等
12.$p^3+q^3=2$求证:$p+q\leq2$
$8 = 4(p + q) = \left( {p + q} \right)[\left( {p + q} \right) + 3\left( {p - q} \right)] \ge \left( {p + q} \right).$
推广:$6\left( {2 - p - q} \right) \geqslant \left( {p + q} \right)\left( {2 - p - q} \right).$
\[LHS - RHS = \left( {p + q - 2} \right)\left( {p + q + 2pq + 4p + 4q + 12} \right) + 3\left( {p + q} \right)\left( {p - q} \right) \geqslant 0\]
13.${(a + b + c - abc)^2} \leqslant (1 + {a^2})(1 + {b^2})(1 + {c^2})$
\[RHS - LHS = {(ab + bc + ca - 1)^2} \geqslant 0\]
14.如果n为正整数,满足$n\nmid 2^n+1,n|2^{2^{n}+1}+1$就称n为好数.
证明:存在质数p>3,使得当$k\ge2$时,$3^k\cdot p$都是好数

TOP

回复 13# hbghlyj
7.代入n=k=4,$\frac{1+4+9+16-2-6-12}6\ne1$
哪里错了

TOP

10.a,b,c,d≥0,a+b+c+d=4,求\(3(a^2+b^2+c^2+d^2)+4abcd\)的范围
最小值为16,当且仅当(1,1,1,1)或\(\frac43,\frac43,\frac43,0\)时取得
hbghlyj 发表于 2019-12-21 22:47

记 `f(a,b,c,d)=3(a^2+b^2+c^2+d^2)+4abcd`,先求最大值,显然 `abcd\leqslant1`,有
\[3(a+b+c+d)^2-f(a,b,c,d)=6\sum_{sym}ab-4abcd\geqslant36\sqrt{abcd}-4abcd\geqslant32abcd,\]得到 `f(a,b,c,d)\leqslant48`,当三个变量为零时取等;

再求最小值,不妨设 `a\geqslant b\geqslant c\geqslant d`,首先显然最小值存在,下面证明取最小值时必然 `a=b=c`,若不然,则 `a>c`,但
\[f(a,b,c,d)-f\left( \frac{a+c}2,b,\frac{a+c}2,d \right)=\frac12(a-c)^2(3-2bd)\geqslant\frac12(a-c)^2\bigl(3-2\sqrt{abcd}\bigr)>0,\]即 `a`, `c` 都变为其平均数时 `f` 更小,矛盾。

所以只需考虑 `g(x)=f(x,x,x,4-3x)`(`x\in[0,4/3]`)的最小值即可,而
\[g(x)-16=4(x-1)^2(x+2)(4-3x)\geqslant0,\]所以最小值为 `16`,当 `x=1` 或 `x=4/3` 取等。

TOP

12.x,y,z≥0,x+y+z=4,求\(\sum \frac x{\sqrt{x+y}}\)的取值范围
最大值\(\frac25\)当且仅当0,3,1时取等
hbghlyj 发表于 2019-12-21 22:47
最大值你写反了,是 5/2,就是 Jack Garfunkel 不等式,可以看看这里:
http://kuing.orzweb.net/redirect ... =5298&pid=26169

至于下界方面 `\displaystyle\sum\frac x{\sqrt{x+y}}>\sum\frac x{\sqrt{x+y+z}}=\sqrt{x+y+z}=2`,当 `x\to4`, `y=z\to0` 时原式 `\to2`,所以范围是 `(2,5/2]`。

TOP

11.x,y,z>0,求\(\sum\frac{yz}{y^2+z^2+3x^2}\)的取值范围
最大值\(\frac35\)当且仅当x:y:z=1:1:1或3:3:2时取等
hbghlyj 发表于 2019-12-21 22:47
这道有点意思,我曾经玩过分母是 `y^2+z^2+2x^2` 时的一个链(《撸题集》P.17 题目 1.1.20),这时是常规取等,不难,没想到现在系数 2 改成 3,竟然就有双取等出现。

由于要顾及双取等,《撸题集》里的证法恐怕用不上,撸了半天,勉强搞了个不太好看还有点暴力的证法:

证明:不妨设 `z=\min\{x,y,z\}`,下面先证明
\[\frac{yz}{y^2+z^2+3x^2}+\frac{zx}{z^2+x^2+3y^2}\leqslant\frac{z(x+y)+\frac8{25}(x-y)^2}{z^2+(x+y)^2}.\quad(*)\]

\begin{align*}
&\frac{yz}{y^2+z^2+3x^2}+\frac{zx}{z^2+x^2+3y^2}-\frac{z(x+y)}{z^2+(x+y)^2}\\
={}&\frac{4xyz(x+y)(x-y)^2}{(y^2+z^2+3x^2)(z^2+x^2+3y^2)\bigl(z^2+(x+y)^2\bigr)}\\
\leqslant{}&\frac{4xyz(x+y)(x-y)^2}{(z^2+4xy)^2\bigl(z^2+(x+y)^2\bigr)},
\end{align*}以及
\[2(z^2+4xy)^2-25xyz(x+y)=(xy-z^2)(7xy-2z^2)+25xy(x-z)(y-z)\geqslant0,\]可知式 (*) 成立。

于是要证原式 `\leqslant3/5`,只需证
\[\frac{z(x+y)+\frac8{25}(x-y)^2}{z^2+(x+y)^2}+\frac{xy}{x^2+y^2+3z^2}\leqslant\frac35,\]去分母可配方为
\[5(x+y-3z)^2(x+y-2z)^2+(x-y)^2(23x^2+110xy+23y^2-50xz-50yz-41z^2)\geqslant0,\]显然成立,即得证。

TOP

16.$x_i≥0,\sum\limits_{i=1}^nx_i=4,$求证$\sum\limits_{cyc}x_1^3x_2≤27$
应该是先证三元再归纳,但三元我也不会证
hbghlyj 发表于 2019-12-24 18:50
先证三元:由轮换对称性,不妨设 `b` 在 `a`, `c` 之间,则
\[\sum a^3b-b(a^3+c^3+abc)=-c(a-b)(b^2-c^2)\leqslant0,\]于是
\[\sum a^3b\leqslant b(a^3+c^3+abc)\leqslant b(a+c)^3=\frac{3b(a+c)^3}3\leqslant\frac13\left( \frac{3b+3(a+c)}4 \right)^4=27.\]
再证 `n`(`n>3`)元,无需归纳:由轮换对称性,不妨设 `x_n` 最大,则有 `x_1^3x_2\leqslant x_1^3(x_2+x_3+\cdots+x_{n-1})` 以及 `x_2^3x_3+x_3^3x_4+\cdots+x_{n-1}^3x_n\leqslant(x_2+x_3+\cdots+x_{n-1})^3x_n`,于是
\[x_1^3x_2+x_2^3x_3+\cdots+x_{n-1}^3x_n+x_n^3x_1\leqslant x_1^3(x_2+x_3+\cdots+x_{n-1})+(x_2+x_3+\cdots+x_{n-1})^3x_n+x_n^3x_1,\]从而由三元时成立可知 `n` 元也成立。

写完才感觉好像以前撸过类似的……算了,懒得查了……

TOP

17.给定正整数n,$\frac1a+\frac1b=1,$求$(a+b)^n-a^n-b^n$的取值范围
a=b=2时取得最大值.下面证$(a+b)^n-a^n-b^n≤2^{2n}-2^{n+1}$
当n=1时显然成立.设n=k时命题成立.由$\frac1a+\frac1b=1,$有ab=a+b≥4,则 $(a+b)^{k+1} =(a+b)^k(a+b)≥ ...
hbghlyj 发表于 2019-12-24 18:50
先说明一下,这个应该要有 a,b 为正数的条件吧?另外,a=b=2 应该是最小值吧?

其实也不用数归:由条件有 `a+b=ab\geqslant4` 及 `(a-1)(b-1)=1`,则
\begin{align*}
(a+b)^n-a^n-b^n&=(ab)^n-a^n-b^n\\
&=(a^n-1)(b^n-1)-1\\
&=(a-1)(a^{n-1}+a^{n-2}+\cdots+1)(b-1)(b^{n-1}+b^{n-2}+\cdots+1)-1\\
&=(a^{n-1}+a^{n-2}+\cdots+1)(b^{n-1}+b^{n-2}+\cdots+1)-1\\
&\geqslant\bigl( \sqrt{a^{n-1}b^{n-1}}+\sqrt{a^{n-2}b^{n-2}}+\cdots+1 \bigr)^2-1\\
&\geqslant(2^{n-1}+2^{n-2}+\cdots+1)^2-1\\
&=(2^n-1)^2-1.
\end{align*}
值得一提的是,这个 `n` 其实也可以不是整数,事实上,当 `u\geqslant1` 时,函数
\[f(t)=\left( \frac1{(0.5+t)^u}-1 \right)\left( \frac1{(0.5-t)^u}-1 \right)\]在 `t\in[0,0.5)` 上递增,证明过程可参考《撸题集》P.434。

另外,也可以用凸函数证明:当 `u\geqslant1` 时,对任意 `a`, `b>0`, `a`, `b`, `ab\ne1` 有
\[\frac{a^u-1}{a-1}\cdot\frac{b^u-1}{b-1}\geqslant\left( \frac{\sqrt{a^ub^u}-1}{\sqrt{ab}-1} \right)^2.\]

\[g(x)=\ln\frac{e^{ux}-1}{e^x-1},\]则只需证明 `g(x)` 下凸即可(虽然 `g(x)` 在 `x=0` 处无定义,但这是可去间断点,所以没所谓),求二阶导数得
\[g''(x)=\frac{e^x}{(e^x-1)^2}-\frac{u^2e^{ux}}{(e^{ux}-1)^2},\]令 `e^x=t^2`, `t>0`,则
\[g''(x)\geqslant0\iff\frac1{(t-t^{-1})^2}\geqslant\frac{u^2}{(t^u-t^{-u})^2},\]由于置换 `t\to t^{-1}` 不改变上式,所以只需证明 `t>1` 的情况,此时即证
\[h(t)=t^u-t^{-u}-u(t-t^{-1})>0,\]求导得
\[h'(t)=ut^{-1}(t^u+t^{-u}-t-t^{-1})\geqslant0,\]所以 `h(t)\geqslant h(1)=0`,即得证。

TOP

本帖最后由 hbghlyj 于 2020-3-15 22:37 编辑

第五组
1.数列$\{x_n\},\{y_n\}:x_0=x_1=1,x_{n+1}=x_n+2x_{n-1},y_0=1,y_1=7,y_{n+1}=2y_n+3y_{n-1}$,证明:除1以外,两个数列中不再有相等的数
$\forall n\in\mathbb N_+,x_{2n}\equiv 3\pmod8,x_{2n+1}\equiv-3\pmod8,y_{2n}\equiv 1\pmod8,y_{2n+1}\equiv-1\pmod8$
2.设r为正整数,数列$\{a_n\}:a_1=1,a_{n+1}=\frac{na_n+2(n+1)^{2r}}{n+2}$,证明:每个$a_n$都是整数,且确定对哪些n,$a_n$是偶数
$(n+2)a_{n+1}=na_{n}+2(n+1)^{2r},(n+1)(n+2)a_{n+1}=n(n+1)a_{n}+2(n+1)^{2r+1}$,构造$\{b_{n}\}:b_{n}=n(n+1)a_{n}$,则$b_n=b_1+\sum\limits_{k=1}^{n-1}(b_{k+1}-b_{k})=2(1+2^{2r+1}+3^{2r+1}+\cdots+n^{2r+1})$,所以$b_n\in\mathbb N,$$\begin{align*}
b_{n}&=2n^{2r+1}+\sum\limits_{k=1}^{n-1}\bigg[k^{2r+1}+(n-k)^{2r+1}\bigg]\\
&=2n^{2r+1}+\sum\limits_{k=1}^{n-1}\bigg[n^{2r+1}-
\binom{2r+1}{1}n^{2r}k+
\binom{2r+1}{2}n^{2r-1}k^2-\cdots+
\binom{2r+1}{2r}n\cdot\,\!k^{2r}\bigg]\\
\end{align*}
$,所以$n|b_n$,$\begin{align*}
b_{n}&=\sum\limits_{k=1}^{n}\bigg[k^{2r+1}+(n+1-k)^{2r+1}\bigg]\\
&=\sum\limits_{k=1}^{n}\bigg[(n+1)^{2r+1}-
\binom{2r+1}{1}(n+1)^{2r}k+
\binom{2r+1}{2}(n+1)^{2r-1}k^2-\cdots+
\binom{2r+1}{2r}(n+1)k^{2r}\bigg]\\
\end{align*}$,所以$n+1|b_n$,所以$n(n+1)|b_n$
若n为偶数,$b_{n}=2n^{2r+1}+2\sum\limits_{k=1}^{\frac n2-1}\bigg[k^{2r+1}+(n-k)^{2r+1}\bigg]$,括号中每一项都是2n的倍数,所以$2n|b_n$,$2|a_n$
若n为奇数,$b_{n}=2\sum\limits_{k=1}^{\frac {n-1}2}\bigg[k^{2r+1}+(n+1-k)^{2r+1}\bigg]+(n+1)\left(\frac {n+1}2\right)^{2r}$,括号中每一项都是2n的倍数,当$\frac {n+1}2$是偶数时,每一项都是2(n+1)的倍数,所以$2(n+1)|b_n$,$2|a_n$;当$\frac {n+1}2$是奇数时,最后一项除以n(n+1)后是奇数,所以$a_n$是奇数
综上所述,$n\equiv 0,2,3\pmod4$时,$a_n$是偶数;$n\equiv 1\pmod4$时,$a_n$是奇数.
3.设$\{a_1,\cdots,a_n\}=\{1,2\cdots n\}$,设f(n)是满足下列条件的数列$\{a_n\}$的个数,
$(1)a_1=1;(2)\abs{a_i-a_{i+1}}\leq2,i=1,2\ldots n-1$,求证:3|f(n)
4.设$S(n)=\sum\limits_{i=1}^np^i,$求证:$f(n)=\frac{(1+p)^n}{2^nS(n)}$在$\mathbb R_+$是凸函数
举p=0.5为例,$f(n)=\frac{(\frac34)^n}{2^{n+1}-2}$
$f'(n)=-\frac{3^n ln2}{2^{ n-1}(2^{n+1} - 2)^2} -\frac {(\frac34)^n ln(\frac43)}{2^{n+1} - 2}$
$f''(n)=\frac{ 3^n ((-11\cdot2^n + 9\cdot4^n + 4) ln^22 + ln3((2^n - 1)^2 ln3 + 2^{n+1}(ln32 - 2^n ln8)) - 4 ln2ln3)}{2^{2 n + 1}(2^{n+1} - 2)^3}$
5.设$f_n(x)=\frac{x^n-1}{x-1}$,a是任意实数,m,n是任意正整数,求方程$f_m(x)=af_n(x)$的实根个数的最大值

TOP

本帖最后由 hbghlyj 于 2020-10-19 16:06 编辑

第六组
2.方程$x^2+x+c=0(c\in\mathbf C)$的两个复数根$\alpha,\beta$满足$\abs\alpha+\abs\beta≤1$,求$\abs{2c-1}$的最大值
${\rm{\alpha }} + {\rm{\beta }} =  - 1,1 = \left| {{\rm{\alpha }} + {\rm{\beta }}} \right| \le \left| {\rm{\alpha }} \right| + \left| {\rm{\beta }} \right| \le 1,\therefore \arg \alpha  = \arg \beta ,\therefore \alpha ,\beta  \in  \mathbf R,\alpha  \le 0,\beta  \le 0,\therefore c = \alpha \beta  \in \mathbf R,{\rm{\Delta }} = 1 - 4c \ge 0,0 \le c \le \frac{1}{4}.\left| {2c - 1} \right| \le 1.$ c=0,α=0,β=-1时等号成立.
3.已知单位复数$z_1,z_2,z_3$满足$Re(z_1\bar{z_2}+z_2\bar{z_3}+z_3\bar{z_1})=-1$,求$\abs{(z_1-z_2)(z_2-z_3)(z_3-z_1)}$的最大值
${\left| {{{\rm{z}}_1} + {{\rm{z}}_2} + {{\rm{z}}_3}} \right|^2} = \sum {z_1}\overline {{z_1}}  + 2Re\left( {\sum {z_1}\overline {{z_2}} } \right) = 3 + 2 \times \left( { - 1} \right) = 1$,设${{\rm{z}}_1},{z_2},{z_3}$的图形为A,B,C,则$1 = {\left| {\overrightarrow {{\rm{OA}}}  + \overrightarrow {{\rm{OB}}}  + \overrightarrow {OC} } \right|^2} = O{A^2} + O{B^2} + O{C^2} + 2\sum \overrightarrow {OA}  \bullet \overrightarrow {OB}  = 3 + 2\sum \cos 2C, - 1 = \sum \cos 2C = 2\sum {\cos ^2}A - 3,\sum {\cos ^2}A = 1,\sum {\cos ^2}A + 2\cos A\cos B\cos C = 1,\therefore \cos A\cos B\cos C = 0,$当且仅当BC=CA=$\sqrt2$时等号成立
4.求所有正整数t,m,n≥3,使得存在无穷多个$a\in\mathbb Z$满足$a^n+a^2-1|a^m+a^t-1$
5.若$a\leq a_i\leq b,\sum\limits_{i=1}^nx_i^2=\sum\limits_{i=1}^ny_i^2=1$,证明$\abs{\sum\limits_{i=1}^na_ix_i^2-\sum\limits_{i=1}^nb_iy_i^2}\leq(b-a)\sqrt{1-\left(\sum\limits_{i=1}^nx_iy_i\right)^2}$
LHS$\leq\max\{a(x-y)+b(y-x),-b(x-y)-a(y-x)\}=(b-a)(x-y)$.只需证$(x-y)^2\leq1-(\sum\limits_{i=1}^nx_iy_i)^2$.不妨设$X=\sum\limits_{i=1}^nx_i^2,Y=\sum\limits_{i=1}^ky_i^2$,化为$1-(\sum\limits_{i=1}^nx_iy_i)^2\geq(\sum\limits_{i=1}^n(x_i-y_i))^2$.由柯西不等式$(\sum\limits_{i=1}^nx_iy_i)^2\leq\sqrt{(\sum\limits_{i=1}^kx_i^2)(\sum\limits_{i=1}^ky_i^2)}+{(\sum\limits_{i=k+1}^nx_i^2)(\sum\limits_{i=k+1}^ny_i^2)}=(\sqrt{(1-x)(1-y)}+\sqrt{xy})^2$
只需证$(x-y)^2\leq1-(\sqrt{xy}+\sqrt{(1-x)(1-y)})^2\Leftrightarrow(x-y)^2+xy+(1-x)(1-y)+2\sqrt{xy(1-x)(1-y)}\leq1\Leftrightarrow2\sqrt{xy(1-x)(1-y)}\leq1\Leftrightarrow2\sqrt{xy(1-x)(1-y)}\leq x(1-x)+y(1-y)$.此即均值不等式.
6.给定整数$n\geq3$,证明:存在n个互不相同的正整数组成的集合S,使得对S的任意两个不同的非空子集A,B,数$\frac{\sum\limits_{x\in A}x}{|A|}$与$\frac{\sum\limits_{x\in B}x}{|B|}$是互素的合数.
7.求7位数A,使1/A的循环节长度最大
A=9999943,循环节长度是9999942
8.对于任意非零实数x,y,设$A=\frac{\abs x}{1+\abs x}+\frac{\abs y}{1+\abs y},B=\frac{\abs{x+y}}{1+\abs{x+y}}$,比较A,B的大小
设$t_1=2-A=\frac1{1+\abs x},t_2=2-B=1+\frac1{1+\abs{x+y}}$,
$t_2=1+\frac1{1+\abs{x+y}}\geq1+\frac1{1+\abs x+\abs y}=:t_3$
设$1+\abs x=m,1+\abs y=n,$则$t_1=\frac1m+\frac1n=\frac{m+n}{mn}=:s_1$,
$t_3=1+\frac1{m+n-1}=\frac{m+n}{m+n-1}:=s_3$,令$S_1=mn,S_3=m+n-1,$则$S_1-S_3=mn-m-n+1=(m-1)(n-1)$,$\because m=|x|+1>1,n=|y|+1>1,\therefore (m-1)(n-1)>0$,即$s_1>s_3$,$\therefore t_1<t_3,$又$\because t_3\leq t_2,\therefore t_1<t_2,\therefore 2-t_1>2-t_2,\therefore A>B$
9.设非零实数x,y,z,求f(x,y,z)=$\frac{|x+y|}{|x|+|y|}+\frac{|y+z|}{|y|+|z|}+\frac{|z+x|}{|z|+|x|}$的取值范围
$\frac{|x+y|}{|x|+|y|},\frac{|y+z|}{|y|+|z|},\frac{|z+x|}{|z|+|x|}\in$[0,1],当且仅当两数互为相反数取0,两数同号取1.故f≤3,例如1,1,1时取等. 三数必有两数同号,对应的项为1,故f≥1,例如1,1,-1时取等.
10.已知正数a,b,a+b都不为1,比较A,B,C的大小
$A=\frac1{log_a2}+\frac1{log_b2},B=2\left(\frac1{log_{a+b}2}-1\right),C=2(1+\frac1{log_{ab}2}-\frac1{log_{a+b}2})$
由换底公式,$A=log_2ab,B=log_2\left(\frac{a+b}2\right)^2,C=log_2\left(\frac2{\frac1a+\frac1b}\right)^2$,由均值不等式$C\leq A\leq B$
11.满足$36^{x^2+y}+36^{y^2+x}=\frac{\sqrt6}3$的实数对(x,y)有几组
$36^{x^2+y}+36^{y^2+x}\geq2\sqrt{36^{x^2+x+y^2+y}}\geq2\cdot6^{-\frac12}$,当且仅当x=y=$-\frac12$时等号成立,故只有一组解.
13.a,b,c>0,$\sum\limits_{\text{sym}}\frac1{2a+b}=1$,求$S=(\sum ab)\left(\frac1{a+b}\right)$的取值范围
$\because \frac {bc}{b+c}\le\frac{b+c}4=\frac{ab+ac}{4a},\therefore \frac{bc}{b+c}\le \sum\frac{\sum ab}{4a+b+c},\therefore a+\frac{bc}{b+c}\le \frac{(2a+b)(2a+c)}{4a+b+c}=\frac1{\frac1{2a+b}+\frac1{2a+c}},\therefore S=\sum\left(a+\frac{bc}{b+c}\right)\le \sum\frac1{\frac1{2a+b}+\frac1{2a+c}}\le\frac9{\sum\limits_{\text{sym}}\frac1{2a+b}}=9$,当a=b=c=2时取等.
因此$S\in (0,9]$
14.a,b,c$\in\mathbf R^+,\sum a=1,$求证:$\sum\frac{ab}{\sqrt{bc+ca}}\leq \frac{\sqrt2}2$
15.设$x_n=\sqrt{2+\sqrt[3]{3+\sqrt[4]{4+\cdots\sqrt[n]n}}}$,n=2,3$\cdots$,证明:对n$\geq4$,$x_{n+1}-x_n<(n!)^{-\frac32}$
设$x_{n,i}=\sqrt[ i ]{i+\sqrt[i+1]{i+1+\cdots\sqrt[n]n}},i=2,3,\cdots,n,y_{n,i}=\sum\limits_{j=0}^{i-1}x_{n+1,i}^jx_{n,i}^{i-1-j},i=2,3,\cdots,n,x_{n+1,i}\ge x_{n,i}\ge i^{\frac1i},y_{n,i}>ix^{i-1}$
16.x,y,z∈[-1,1],$x^2+y^2+z^2=1+2xyz$,求证对正整数n都有$x^{2n}+y^{2n}+z^{2n}\le1+2x^ny^nz^n$
17.求最小的实数$\mu,$使得任意正数$a_1,a_2,\cdots,a_n$有
$a_1a_2+a_2a_3+\cdots+a_{n-1}a_n\le\mu(a_1^2+a_2^2+\cdots+a_n^2)$
解:令$\lambda$为待定常数(与n有关).取实数$\lambda_1,\lambda_2,\cdots,\lambda_n$使得$\lambda_1=\lambda,\lambda_{k+1}=\lambda-\frac1{4\lambda_k}(k=1,2,\cdots,n-1),\lambda_n=0$.
由$\{\lambda_k\}_{k=1}^n$的递推式可知$\frac{\lambda_{k+1}-x_1}{\lambda_{k+1}-x_2}=\frac{\lambda-x_1}{\lambda-x_2}\cdot\frac{\lambda_k-x_1}{\lambda_k-x_2}$
这里$x_1,x_2$为特征方程$x=\lambda-\frac1{4x}$的两根.即若令$\lambda=\cos\theta$,则$x_1=\frac{\lambda+\sqrt{\lambda^2-1}}2=\frac12e^{i\theta},x_2=\frac{\lambda-\sqrt{\lambda^2-1}}2=\frac12e^{-i\theta}$
于是,$\frac{x_1}{x_2}=\frac{\lambda_n-x_1}{\lambda_n-x_2}=\left(\frac{\lambda-x_1}{\lambda-x_2}\right)^{n-1}\cdot \frac{\lambda_1-x_1}{\lambda_1-x_2}=\left(\frac{x_2}{x_1}\right)^n$
可得$1=\left(\frac{x_1}{x_2}\right)^{n+1}=e^{i\cdot2(n+1)\theta}\ldots\ldots(1)$
为迎合式(1),取$\theta=\frac\pi{n+1},$即$\lambda=\cos\frac\pi{n+1}$.
$a_1a_2+a_2a_3+\cdots+a_{n-1}a_n\le\left(\lambda_1a_1^2+\frac1{4\lambda_1}a_2^2\right)+\left(\lambda_2a_2^2+\frac1{4\lambda_2}a_3^2\right)+\cdots+\left(\lambda_{n-1}a_{n-1}^2+\frac1{4\lambda_{n-1}}a_{n}^2\right)=\lambda(a_1^2+a_2^2+\cdots+a_n^2).$
等号显然能成立.故所求$\mu=\cos\frac\pi{n+1}$
注:若只为迎合式(1),当然也可以取$\theta=\frac{2\pi}{n+1},\frac{3\pi}{n+1},\cdots$,但这样就不能使$\lambda_1,\lambda_2,\cdots,\lambda_{n-1}$全不为0.而实际上,从$\{\lambda_k\}_{k=1}^n$的递推式看,为使之有意义,必须有$\lambda_1,\lambda_2,\cdots,\lambda_{n-1}\ne0$
18.$f(x)=20 x^3-3 x^2-10 x^2 \log (x)-36 x^2 \sqrt{x-\log (x)}+4 x-4 x \log ^2(x)-4 x \log (x)+12 x \log (x) \sqrt{x-\log (x)}+18 x \sqrt{x-\log (x)}-3$,求证$(x-1)f(x)>0$
19.已知函数$f: \mathbb{Z}^{+} \rightarrow \mathbb{Z}^{+}$是单调非减函数,对任意的互质正整数$m,n$,均满足$f(m,n)=f(m)f(n).$试比较$f(2) f(8)  $和$  f(3) f(5)  $的大小.
\begin{array}{l}
f(2) \cdot f(191)>f(3) \cdot f(121) \\
f(8) \cdot f(121)>f(5) \cdot f(191) \\
\Rightarrow f(2) \cdot f(8)>f(3)f(5)
\end{array}

TOP

回复 21# hbghlyj

大咖可以做成一个专集了。

TOP

本帖最后由 hbghlyj 于 2020-5-14 09:56 编辑

第七组.
1.给定整数n$\geq2$,实数$x_1,x_2,\cdots,x_n\in[0,1]$,证明:$\sum\limits_{1\leq k < j\leq n}kx_kx_j\leq\frac{n-1}3\sum\limits_{k=1}^nkx_k$
法①当n=2时,ab$\le\frac13a+\frac23b\Leftrightarrow\left( {a - \frac{1}{3}} \right)\left( {b - \frac{2}{3}} \right) \leqslant \frac{1}{3} \bullet \frac{2}{3}$,当且仅当a=1,b=0时取等.
$kx_kx_j\le\frac k3x_k+\frac{2k}3x_j$,两边对$1\le k<j\le n$求和得???
法②由于两边关于$x_k$是一次式,取最值时$x_k$=0或1,设$x_{a_1}=x_{a_2}=\cdots=x_{a_m}=1$,其余$x_i$为0.
$(m-1)a_1+(m-2)a_2+\cdots+a_{m-1}\le\frac{n-1}3(a_1+a_2+\cdots+a_n)$.设m-i-1$\le\frac{n-1}3\le m-i,\therefore$可认为$a_1,a_2,\cdots,a_m$为连续m个正整数,???
2.$n\in\mathbb N_+,a_1,a_2,\cdots,a_n\in\mathbb R,r_1,r_2,\cdots,r_n\in\mathbb R_+,$证明:$\sum\limits_{i=1}^n\sum\limits_{j=1}^na_ia_j\min(r_i,r_j)\geq0$
由于对任意i,j,交换$a_i,a_j$不等式不变,不妨设$r_1\ge r_2\ge \cdots\ge r_n,$左式=$r_1a_1^2+r_2(2a_1a_2+a_2^2)+r_3(2a_1a_3+2a_2a_3+a_3^2)+\cdots=(r_1-r_2)a_1^2+(r_2-r_3)(a_1^2+2a_1a_2+a_2^2)+(r_3-r_4)(a_1^2+2a_1a_2+a_2^2+2a_1a_3+2a_2a_3+a_3^2)+\cdots+(r_{n-1}-r_n)(a_1^2+\cdots+a_{n-1}^2)+r_n(a_1^2+\cdots+a_n^2)\geq0$
令$f_i(x)=\led
{a_i}\quad 0 \le x \le r \\
0\quad \text{otherwise}
\endled$
$f_i(x) \bullet f _j(x) =\led \begin{array}{l}
{a_i}{a_j}\end{array}\quad 0 \le x \le \min \{ {r_i},{r_j}\} \\
0\quad \text{otherwise}\endled$
$F(x) = \mathop \smallint \nolimits_0^{ + \infty } {f_i}(x){f_j}(x){\rm{d}}x = {f_1}(x) + {f_2}(x) +  \cdots  + {f_n}(x) = \mathop \smallint \nolimits_0^{ + \infty } {(F(x))^2}{\rm{d}}x > 0$
3.切比雪夫不等式的加强:若一个实数列满足:从第二项起,每项均不大于前面所有项的算术平均值,则称这个实数列是伪减的.已知两个数列$\{a_n\},\{b_n\}$是伪减的,则$\sum\limits_{i=1}^na_ib_i\geq\frac1n\sum\limits_{i=1}^na_i\sum\limits_{i=1}^nb_i$
对n归纳,n=1时为恒等式,n=2时.设对n成立,下证对n+1成立:
$\sum\limits_{i = 1}^{n + 1} {{a_i}} {b_i} \ge \frac{1}{n}\sum\limits_{i = 1}^n {{a_i}} \sum\limits_{i = 1}^n {{b_i}}  + {a_{n + 1}}{b_{n + 1}}$.
4.由正整数组成的有限集A满足:A的任意两个不同的子集的所有元素之和均不同,证明:A的所有元素的倒数之和小于2
5.设整数n$\geq$3,$a_1,a_2,\cdots,a_n$是实数,S是{1,2$\cdots$n}的一个非空子集,证明$\sum\limits_{i\in S}\leq\sum\limits_{1\leq i\leq j\leq n}(a_i+\cdots+a_j)^2$
6.设$x_1,x_2,\cdots,x_n\in\mathbf R$,证明:$\sum\limits_{i=1}^n\sum\limits_{j=1}^n\abs{x_i+x_j}\geq n\sum\limits_{i=1}^n\abs{x_i}$
7.设整数n$\geq$2,正实数$x_1,x_2\cdots x_n,y_1,y_2\cdots y_n$满足$x_1x_2\cdots x_n=y_1y_2\cdots y_n=1$,且对任意$1\leq j\leq n$都有$(y_i-1)(y_i-x_i)\leq0$.证明$x_1+x_2+\cdots+x_n\geq y_1+y_2+\cdots+y_n$
8.设整数n$\geq$4,正实数$x_1,x_2,\cdots,x_n$之积为1,证明$\sum\limits_{i=1}^n\frac{x_i^2}{(x_i+1)^2}\geq1$
9.给定整数$n\geq2,$求最小的常数d,使得不等式$\sum\limits_{i=1}^n\frac1{a_i^2+d}\leq\frac n{1+d}$对任意和为n的非负实数$a_1,a_2,\cdots,a_n$均成立.
10.设$a_1,a_2,\cdots,a_{10}$和$b_1,b_2,\cdots,b_{10}$是两个1,2,$\cdots$,10的排列,将$a_1b_1,a_2b_2,\cdots,a_{10}b_{10}$从大到小排成一列$M_1\geq M_2\geq\cdots\geq M_{10}$,求$M_1,M_2$的最值
11.设$a_1,a_2,\cdots,a_n$是一些实数,证明
(1)存在$\varepsilon_1,\varepsilon_2\cdots\varepsilon_n\in\{-1,1\}$,使得$\sum\limits_{i=1}^n\varepsilon_ia_i\leq\max\limits_{1\leq i\leq n}\abs{a_i}$
(2)存在不全为0的$\varepsilon_1,\varepsilon_2\cdots\varepsilon_n\in\{-1,0,1\}$,使得$\sum\limits_{i=1}^n\varepsilon_ia_i\leq\frac{\sum\limits_{i=1}^n\abs{a_i}}{2^n-1}$
12.设整数n$\geq3$,非负实数$a_1\geq a_2\geq\cdots\geq a_n$,将它们排在一个圆周上,证明
(1)对任意$1\leq i\leq\frac n2$,圆周上存在相邻两数之积不小于$a_ia_{n-i}$
(2)存在一种排列方式,使得圆周上任意相邻两数之积不大于$\max\limits_{1\leq i\le \frac n2}a_ia_{n-i}$
13.给定整数$n\geq3$,求最大的整数K=K(n),使得对任意n元实数组$(x_1,x_2,\cdots,x_n)$只要其任一排列$y_1,y_2,\cdots,y_n$满足$\sum\limits_{i=1}^{n-1}y_iy_{i+1}\geq-1$,就有$\sum\limits_{1\leq i\le j\leq n}x_ix_j\geq K$.
14.给定整数$n\geq5$,实数$x_1,x_2\cdots x_n$满足$-1\leq x_i\leq 1$(i=1,2$\cdots$n)及$\sum\limits_{i=1}^nx_i=0$.对$x_1,x_2\cdots x_n$的任一排列$y_1,y_2,\cdots,y_n$,计算$y_1y_2+y_2y_3+\cdots+y_{n-1}y_n+y_ny_1$的值,并设这些值中最大的一个为A,求A的最小可能值
15.设正整数$a_1,a_2,\cdots,a_{31},b_1,b_2,\cdots,b_{31}$满足
(1)$1\leq a_1\le a_2\le\cdots\le2015,1\leq b_1\le b_2\le\cdots\le b_{31}\leq 2015$,
(2)$a_1+a_2+\cdots+a_{31}=b_1+b_2+\cdots+b_{31}$,
求$\sum\limits_{i=1}^n\abs{a_i-b_i}$的最大可能值
16.$n\in\mathbf N_+,a_1,a_2,\cdots,a_n,b_1,b_2,\cdots,b_n,A,B\in\mathbf N_+,a_i\le b_i,a_i\le A,i=1,2,\cdots,n,\frac{b_1b_2\cdots b_n}{a_1a_2\cdots a_n}\leq\frac BA$,证明$\frac{(b_1+1)(b_2+1)\cdots(b_n+1)}{(a_1+1)(a_2+1)\cdots(a_n+1)}\le\frac{B+1}{A+1}$
$k_i=\frac{b_i}{a_i}\ge1,i=1,2,\cdots,n,$记$\frac BA=K$,条件化为$k_1k_2\cdots k_n\le K,$要证$\prod\limits_{i=1}^n\frac{k_ia_i+1}{a_i+1}\le\frac{KA+1}{A+1}.$对i=$1,2,\cdots,n,$由于$k_i\ge1$及$0<a_i\leq A$知,$\frac{k_ia_i+1}{a_i+1}=k_i-\frac{k_i-1}{a_i+1}\leq k_i-\frac{k_i-1}{A+1}=\frac{k_iA+1}{A+1}$.结合$K\ge k_1k_2\cdots k_n$知,只需证$A>0,k_i\ge1(i=1,2,\cdots,n)$时,有$\prod\limits_{i=1}^n\frac{k_iA+1}{A+1}\le\frac{k_1k_2\cdots k_nA+1}{A+1}.$
对n进行归纳.当n=1时,结论显然成立.当n=2时,由A>0,$k_1,k_2\ge1$可知$\frac{k_iA+1}{A+1}=\left(\prod\limits_{i=1}^m\frac{k_iA+1}{A+1}\right)\le\frac{k_1k_2\cdots k_mA+1}{A+1}\bullet\frac{k_{m+1}A+1}{A+1},$
17.不存在f:$\mathbf R^+\to\mathbf R^+$,使得$\left(f(x)\right)^2\ge f(x+y)\left(f(x)+y\right)$对任意x,y>0成立
18.对于固定的正整数s,t(s<t),设$a_n=\sum\limits_{i = s}^t {C_n^i} $,证明:对于所有正整数n,均有$a_n^2 \geqslant {a_{n - 1}}{{\text{a}}_{n + 1}}$
19.a,b,c$\in[0,1],a+b,b+c,c+a\ge1,$求证:$1-\frac{2\sqrt2abc}{\sqrt{a^2+b^2+c^2}}\leq(1-a)^2+(1-b)^2+(1-c)^2\leq1.$
20.设$a_1\geq a_2\geq\cdots\ge a_n>0$,给定k$\in\{1,2,\cdots,n\},$证明:可将$a_1,a_2,\cdots,a_n$分成k组,使得每组的所有数之和均不小于$\frac12\mathop {\min }\limits_{1 \leqslant j \leqslant k} \frac{{\sum\limits_{i = j}^n {{a_i}} }}{{k + 1 - j}}$
21.求所有正整数n>1,使得存在两个不是整数的有理数a,b满足$a+b,a^n+b^n\in\mathbf Z$
n为奇数,取$a=\frac13,b=\frac{3^n-1}3$满足条件;下证n为偶数时不存在这样的a,b:
$a=\frac xy,b=\frac zw,(x,y)=(z,w)=1,y,w>1,a+b\in\mathbf Z\Rightarrow y=w\left(\frac{xw+yz}{yw}\in\mathbf Z\Rightarrow y\mid xw\Rightarrow y\mid w\Rightarrow w\mid yz\Rightarrow w\mid y\Rightarrow y=w\right)$.
$z=ky-x,y^n\mid x^n+(ky-x)^n,x^n+(ky-x)^n=My^2-nkyx+2x^n\Rightarrow y\mid 2x^n\Rightarrow y|2\Rightarrow y=2\Rightarrow 2|x+1$,而$4|2x^n$,矛盾
22.对任给正整数m,n,总存在正整数k,使得$2^k-m$至少有n个不同的素因子
对n归纳,只需考虑m为奇数的情况.n=1时,$k_1=3m,2^{3m}-m>1$.假设n时成立,存在$k_n$使$A_n=2^{k_n}-m$至少含n个不同的素因子.当n+1时,$2^{k_n+\phi(A_n^2)}-m\equiv2^{k_n}-m\equiv A_n\pmod{A_n^2}$,令$k_{n+1}=k_n+\phi(A_n^2),A_n\mid2^{k_{n+1}}-m,A_n\not\mid\frac{2^{k_{n+1}-m}}{A_n},\frac{2^{k_{n+1}-m}}{A_n}\equiv1\pmod{A_n},$取$\frac{2^{k_{n+1}-m}}{A_n}$的质因数p,p与$A_n$中
23.非负数a,b,c,d和为100,求\[S = \sqrt[3]{{\frac{a}{{b + 7}}}} + \sqrt[3]{{\frac{{\text{b}}}{{c + 7}}}} + \sqrt[3]{{\frac{c}{{d + 7}}}} + \sqrt[3]{{\frac{d}{{a + 7}}}}\]的最大可能值
24.设m,n$\ge2$是整数,$f(x_1,x_2,\cdots,x_n)$是一个实系数多项式,$f(x_1,x_2,\cdots,x_n)=\lfloor\frac{x_1+x_2+\cdots+x_n}n\rfloor$对任意$x_1,x_2,\cdots,x_n\in\{0,1,\cdots,m-1\}$均成立.证明$\deg f\geq n$
25.互异正整数m,n,p,q满足m+n=p+q,$\sqrt m+\sqrt[3]n=\sqrt p+\sqrt[3]q\ge2004$,求[m,n,p,q]的最小值
令$m=a^2,n=b^3,p=c^2,q=d^3$,则$a+b=c+d\Rightarrow a-c=d-b,a^2+b^3=c^2+d^3\Leftrightarrow(a+c)(a-c)=(d-b)(d^2+bd+d^2)\Rightarrow a+c=d^2+bd+d^2$,令$a-c=d-b=1,b>2004\Rightarrow a=\frac{b^2+bd+d^2+1}2,c=\frac{b^2+bd+d^2-1}2\Rightarrow a,c\in\mathbf Z$
a=1954,b=62,c=2015,d=1,m=3818116,n=238328,p=4060225,q=1,[m,n,p,q]=961149422238200
这好像不能说明最小解的最小公倍数是最小的
26.$\sigma(n)$表示正整数n的正约数之和,证明:任给$\varepsilon>0$,存在无穷多个正整数n使$\frac{f(n)}n>\varepsilon$
令n=$k!$,$f(n)>n\left(1+\frac12+\cdots+\frac1k\right)$,$\frac{f(n)}n>1+\frac12+\cdots+\frac1k$,当$k\to\infty$时$RHS\to\infty$.
$任给$\varepsilon>0$,是否存在无穷多个正整数n使$\frac{f(n)}{n^2}>\varepsilon$
27.设m为正整数,素数p>m,证明:使得$m^2+n^2+p^2-2mn-2np-2pm$为完全平方数的正整数n的个数与p无关.
$m^2+n^2+p^2-2mn-2np-2pm=(m+n-p)^2-4mn$为完全平方数,$mx^2-(m+n-p)x+n$可分解为$(d_1x+a)(d_2x+b),d_1,d_2\in\mathbf N_+,d_1d_2=m,ab=n,-(ad_2+bd_1)=m+n-p,p=d_1d_2+ab-ad_2-bd_1=(d_1-a)(d_2-b),\{d_1-a,d_2-b\}=\{1,p\}$或$\{-1,-p\}$,$d_1+d_2-a-b=1+p>1+m=1+d_1d_2\ge d_1+d_2,d_1+d_2-a-b=-(1+p)<-(1+m)=1+d_1d_2\ge d_1+d_2$
$(m+n-p)^2-4mn=(2m+d_1p+d_2)^2-4m(m+p+d_1p+d_2)=4m^2+4m(d_1p+d_2)-4m^2-4m(p+d_1p+d_2)+(d_1p+d_2)^2=(d_1p-d_2)^2$
28.设m与n互素,证明:$m^{φ(n)}+n^{φ(m)}≡1\pmod{mn}$
由欧拉定理,$m^{φ(n)}+n^{φ(m)}≡n^{φ(m)}≡1\pmod{m}$,同理$m^{φ(n)}+n^{φ(m)}≡1\pmod{n}$,故得证.
29.设0≤a,b,c≤1, 求证$\sum\frac a{1+bc}≤2$
1.0≤u,v≤1=>0≤(1-u)(1-v)=>u+v-1≤uv
2.a+b+c≤2,a/(bc+1)+b/(ac+1)+c/(ab+1)≤a+b+c≤2
3.a+b+c-2≤(b+c-1)a≤abc.
ⅱ)a+b+c=a/(bc+1)+b/(ac+1)+c/(ab+1)+abc[1/(bc+1)+1/(ac+1)+1/(ab+1)]
≥a/(bc+1)+b/(ac+1)+c/(ab+1)+
[a+b+c-2][1/(1+1)+1/(1+1)+1/(1+1)]=
=a/(bc+1)+b/(ac+1)+c/(ab+1)+3/2[a+b+c-2]
==>
a/(bc+1)+b/(ac+1)+c/(ab+1)≤
a+b+c-3/2(a+b+c-2)=2-1/2(a+b+c-2)<2.
30.设0≤a,b,c≤1,证明:$\sum a{b+c+1}+(1-a)(1-b)(1-c)≤1$
31.若$x^3+y^3=2$,求证:x+y≤2
法①假设x+y>2,x³+y³ > x³+(2-x)³ = 6x²-12x+8 = 6(x-1)²+2 ≥2
法②$\because x^3+y^3=(x+y)(x^2-xy+y^2)>0,x^2-xy+y^2>0,\therefore x+y>0,\because(x+y)(x^3+y^3)=x^4 + y^4 + x^3y + xy^3≥x^4 + y^4 + 2x^2y^2=(x^2+y^2)^2≥\frac14(x+y)^4,\therefore x+y\le 2$
32.$x_{ij}>0,\frac1{\sum\limits_{i=1}^m\frac1{\sum\limits_{j=1}^nx_{ij}}
}\ge\sum\limits_{j=1}^n\frac1{\sum\limits_{i=1}^m\frac1{x_{ij}}}$
这个不等式的物理意义是添加导线后电阻不增加
新建位图图像.gif
2020-5-14 09:07

先证m=2的情形,即$\frac1{\frac1{\sum\limits_{j=1}^nx_{1j}}+\frac1{{\sum\limits_{j=1}^nx_{2j}}}}\ge\sum\limits_{j=1}^n\frac1{\frac1{x_{1j}}+\frac1{x_{2j}}}$
$\Leftrightarrow\frac{\sum\limits_{j=1}^nx_{1j}{\sum\limits_{j=1}^nx_{2j}}}{\sum\limits_{j=1}^nx_{1j}+{\sum\limits_{j=1}^nx_{2j}}}\ge\sum\limits_{j=1}^n\frac{x_{1j}x_{2j}}{x_{1j}+x_{2j}}$
$\Leftrightarrow\sum\limits_{j=1}^nx_{1j}-\frac{\left(\sum\limits_{j=1}^nx_{1j}\right)^2}{\sum\limits_{j=1}^nx_{1j}+{\sum\limits_{j=1}^nx_{2j}}}\ge\sum\limits_{j=1}^nx_{1j}-\sum\limits_{j=1}^n\frac{x_{1j}^2}{x_{1j}+x_{2j}}$
$\Leftrightarrow \frac{\left(\sum\limits_{j=1}^nx_{1j}\right)^2}{\sum\limits_{j=1}^nx_{1j}+{\sum\limits_{j=1}^nx_{2j}}}\le\sum\limits_{j=1}^n\frac{x_{1j}^2}{x_{1j}+x_{2j}}$
此即柯西不等式,当且仅当$x_{1j}:x_{2j}=\text{const}$时等号成立
假设命题对m为真,
$\frac1{\sum\limits_{i=1}^{m+1}\frac1{\sum\limits_{j=1}^nx_{ij}}
}\ge\frac1{\frac1{\sum\limits_{j=1}^n\frac1{\sum\limits_{i=1}^m\frac1{x_{ij}}}}+\frac1{\sum\limits_{j=1}^nx_{m+1,j}}
}\ge\sum\limits_{j=1}^n\frac1{\sum\limits_{i=1}^{m+1}\frac1{x_{ij}}}$(第二个$\ge$是根据m=2时的不等式),因此命题对m+1也为真,由归纳定理得证。当且仅当$x_{1j}:x_{2j}:\cdots:x_{mj}=\text{const}$时等号成立。

TOP

本帖最后由 hbghlyj 于 2020-5-15 21:29 编辑

第八组
1.求常数m的最大值,使得方程组$\left\{ \begin{array}{l}
x + y = z + u\\
2xy = zu
\end{array} \right.$的任意正整数解(x,y,z,u),当$x\geq y$时有m$\leq \frac xy$
2.$a,b,c,d\in\mathbf Z$,a>b>c>d,ac+bd=(b+d+a-c)(b+d-a+c),证明:ab+cd是合数
下面三个证明在标红处使用了a,b,c,d>0
法①假设p=ab+cd是素数,ac+bd=(b+d+a-c)(b+d-a+c)=$(b+d)^2-(a-c)^2$,得$a^2-ac+c^2=b^2+bd+d^2$,将$a=\frac{p-cd}b$代入,化简得$p(p-2cd-bc)=(b^2-c^2)(b^2+bd+d^2)$,因为$p=ab+cd$>ab>$b^2>b^2-c^2>0$,p是素数得$(p,b^2-c^2)=1, p|b^2+bd+d^2$,又$2p>2(b^2+d^2)>(b+d)^2>b^2+bd+d^2$,这说明$p=b^2+bd+d^2$,于是$ab+cd=b^2+bd+d^2$,即b(a-b)=d(b+d-c),由于b>c>d,所以b>c-d>0,这样(b,d)>1,意味着(b,d)|p,p不是素数,矛盾
法②假设ab+cd是质数,ab+cd=(a+d)c+(b-c)a=m(a+d,b-c),m$\in {{\mathbb{N}}_{+}}$.所以m=1或(a+d,b-c)=1
1°m=1,则(a+d,b-c)=ab+cd>ab+cd-(a-b+d+c)=(a+d)(c-1)+(b-c)(a+1)(a+d,b-c),矛盾
2°(a+d,b-c)=1,则(a+d)b-(b-c)a=ac+bd=(b+d+a-c)(b+d-a+c),(a+d)(a-c-d)=(b-c)(b+c+d),存在正整数k使得a-c-d=k(b-c),b+c+d=k(a+d)将这两式相加可得:a+b=k(a+b-c+d),
而k(c-d)=(k-1)(a+b)若k=1,则c=d,此与a>b>c>d矛盾!若k≥2,则$2≥\frac k{k-1}=\frac{a+b}{c-d}>2$矛盾.
法③因为(ab+cd)-(ac+bd)=(a-d)(b-c)>0,所以ab+cd>ac+bd;又因为(ac+bd)-(ad+bc)=(a-b)(c-d)>0,所以ab+cd>ac+bd>ad+bc.
由于ac+bd=(a+b-c+d)(-a+b+c+d),所以$a^2-ac+c^2=b^2+bd+d^2$.
因此$(ac+bd)(b^2+bd+d^2)=ac(b^2+bd+d^2)+bd(a^2-ac+c^2)=acb^2+acd^2+a^2bd+bc^2d
=(ab+cd)(ad+bc)$,所以(ab+cd)(ad+bc)|(ac+bd).
假设ab+cd是一个素数,由于ab+cd>ac+bd>1,所以(ab+cd,ac+bd)=1,因此ad+bc |ac+bd,与ac+bdad+bc矛盾.所以ab+cd是一个合数.
法④$a^2-ac+c^2=b^2+bd+d^2,(2a-c)^2+3c^2=(2b+c)^2+3d^2,3(c-d)(c+d)=(2b+d+2a-c)(2b+d-2a+c),$设$c-d=pq,c+d=mn,2b+d+2a-c=nq,2b+d-2a+c=3mp,$则$c=\frac{mn+pq}2,d=\frac{mn-pq}2,b=\frac{nq+3mp+pq-mn}4,a=\frac{nq+pq-3mp+mn}4,n>3p,c>b\Leftrightarrow\frac{(n-p)(q-3m)}4>0,ab+cd=\frac1{16}(3m^2+q^2)(n-p)(n+3p),b\in\mathbf Z,4\mid(n+p)(m-q)$
1°4|n-p,n=3p=1$\Rightarrow$2|m-q,4|$3m^2+q^2$
2°2|n-p,4$\not\mid$n-p,$3m^2+q^2>4\Rightarrow n-p=2,n>3p$
3°2|n+p+1,ad+bc=$\frac14mq(3p^2+n^2),$ac+bd=$\frac14m(n-p)(3n+p)q$.
3.$x,y,z\geq 0,xy+yz+zx=1,$求证$\frac1{\sqrt{x+y}}+\frac1{\sqrt{y+z}}+\frac1{\sqrt{z+x}}\geq2+\frac{\sqrt2}2$
4.a,b,c,d$\ge$0,$b+c\geq a+d$,求$\frac b{c+d}+\frac c{a+b}$的取值范围
最小值为$\frac{1}{2} \left(2 \sqrt{2}-1\right)$,a=0,b=1,c=$\frac{1}{2} \left(\sqrt{2}-1\right)$,d=$\frac{1}{2} \left(\sqrt{2}+1\right)$
最大值为$\infty$,c=d=0或a=b=0.
5.正数列$\{a_n\}$满足${{a}_{n}}=\frac{a_{n+1}^{2}}{n}+{{a}_{n+1}},n\ge 1$.求证$\mathop {\lim }\limits_{n \to \infty } ({x_n} - 2{x_{n + 1}} + {x_{n + 2}}) = 0$
6.有界实数列$\{x_n\}$满足$\mathop {\lim }\limits_{n \to \infty } ({x_n} - 2{x_{n + 1}} + {x_{n + 2}}) = 0$,证明$\mathop {\lim }\limits_{n \to \infty } ({x_n} - {x_{n + 1}}) = 0$
7.x+y+z=xy+yz+zx,求$\sum\frac x{\sqrt{x^4+x^2+1}}$的最小值
$f(x)=\frac x{\sqrt{x^4+x^2+1}}$为奇函数,值域为$\left[-\frac1{\sqrt3},\frac1{\sqrt3}\right]$,在$(-\infty,-1)$递减,$(-1,1)$递增,$(1,+\infty)$递减.容易得到3f(0)=0,故f(x)+f(y)+f(z)的最小值不超过0.故求最小值我们只考虑x,y,z中有负数的情况即可
1°若x,y,z中有两个非负,一个非正,不妨设$x\ge0,y\ge0,z\le0$,则$f(x)+f(y)+f(z)\ge0+0-\frac1{\sqrt3}=-\frac1{\sqrt3}$.讨论可得不能取等.
2°x,y,z不全是负数,若两负一正,不妨设x>0,y<0,z<0.由题意xy+yz+zx=x+y+z$\Leftrightarrow \frac1{xy}+\frac1{yz}+\frac1{zx}=\frac 1x+\frac1y+\frac1z,$且$f(x)=f\left(\frac1x\right)$,不妨设$x\ge1.$此时$yz-y-z=x(1-y-z)\ge1-y-z,$即$yz\ge1$.假设$y\le z$则$y\le-1$.此时$x+y=\frac{xy-z}{1-z}\le\frac{y-z}{1-z}\le0$,则$1\le x\le -y,$结合函数f(x)在$[1,+\infty)$单调递减,$f(x)\ge f(-y),f(x)+f(y)\ge0,$此时$f(x)+f(y)+f(z)\ge-\frac1{\sqrt3}$,当且仅当x=1,y=z=-1时取等
综上所述,原式最小值为$-\frac1{\sqrt3}$
8.实数列$\{a_n\}:a_0=0,a_1=1,$对任意$n\geq2$,存在$1\leq k\leq n$使得$a_n=\frac{a_{n-1}+\cdots+a_{n-k}}k.$求$a_{2018}-a_{2017}$的最大可能值
9.$a_1,a_2,\cdots,a_n\geq1$,证明$\sum\limits_{i=1}^n\sqrt{a_i^{2n}-a_i^{2(n-1)}}\leq\left(\prod\limits_{i=1}^na_i\right)^n$
10.求所有整数对(p,q),使得$x^4+2px^2+qx+p^2-36$有四个整数根(可以相同)
11.f:$\mathbf R\to[0,+\infty)$满足,对任意x,y$\in\mathbf R$,均有$f^2(x+y)+f^2(x-y)=2f^2(x)+2f^2(y),$证明:对任意x,y$\in\mathbf R$,均有f(x+y)≤f(x)+f(y)
12.$\frac{x}{{w + 2}} + \frac{y}{{w + 1}} + \frac{z}{w} = 0,w > 0,x \ne 0,$证明:关于t的方程$xt^2+yt+z=0$在(0,1)内有解
13.$a,b,c,d>0,\frac{a}{{1 + a}} + \frac{b}{{1 + b}} + \frac{c}{{1 + c}} + \frac{d}{{1 + d}} = 1$,求abcd的最大值
14.对给定的正整数n,求
(1)$\mathop \sum \limits_{i = 1}^{n - 1} i \bullet \sin \frac{{2i\pi }}{n}$
(2)$\mathop \sum \limits_{i = 1}^{n - 1} i \bullet \cos \frac{{2i\pi }}{n}$
15.求方程${z^6} + {z^4} + {z^3} + {z^2} + 1 = 0$的虚部为正的根之积
\[z = \cos {84^\circ } + i\sin {84^\circ }\]
16.$\alpha ,\beta ,\gamma ,\delta  \in \left( {0,\frac{\pi }{2}} \right),\alpha  + \beta  + \gamma  + \delta  = \pi ,$证明\[\frac{{\tan \alpha  - 1}}{{\tan \alpha  + 1}} + \frac{{\tan \beta  - 1}}{{\tan \beta  + 1}} + \frac{{\tan \gamma  - 1}}{{\tan \gamma  + 1}} + \frac{{\tan \delta  - 1}}{{\tan \delta  + 1}} \ge 0\]
17.$x,y,z \in {{\bf{R}}^ + },a = {x^2}{(y - z)^4},b = {y^2}{(z - x)^4},c = {z^2}{(x - y)^4},$证明\[{\left( {a + b + c} \right)^{\frac{3}{2}}} \ge 2\left( {a\sqrt b  + b\sqrt c  + c\sqrt a } \right)\]
18.${a_1},{a_2}, \cdots ,{a_n} \in {\bf{R}},$证明\[\mathop \sum \limits_{k = 1}^{n + 1} {({a_k} - {a_{k - 1}})^2} \le 2\left( {1 + \cos \frac{\pi }{{n + 1}}} \right)\mathop \sum \limits_{k = 1}^n a_k^2\],其中${a_0} = {a_{n + 1}} = 0$
19.对无穷集A,若存在映射$f:\mathbf R\to A,g:\mathbf R\to \mathbf R,\forall x_1,x_2,|f(x_1)-f(x_2)|+|g(x_1)-g(x2)|>1.$则称A为一个“好集合”,(1)证明: 存在无穷多个“好集合”: (2)对正实数x,若存在正整数k < 2020使得x为一个正有理数,则称x为“合格的”.集合X为所有“合格的"的实数所构成的集合。试问X是否为一个“好集合”?证明你的结论。
(3)求最大的正实数C,使得对任意无穷正实数数列$\{a_n\}_{n\ge0}$,不等式$a_n +1>a_{n-1}\cdot\sqrt[n]C$都对无穷多个正整数n成立
20.求值\[\mathop \sum \limits_{k = 1}^{2020} {\left( {\cos \frac{{2k - 1}}{{4041}}\pi } \right)^{2019}}\]
这帖3#的结论5\[\mathop \prod \limits_{k = 1}^{2020} \sin \frac{{k\pi }}{{4041}} = \frac{{3\sqrt {449} }}{{{2^{2020}}}}\]
21.$a,b,c \in {{\bf{R}}^ + }.$求证:\[\sum {\sqrt {\frac{a}{{7a + b + c}}} }  \le 1\]
22.设正实数$a_i(i=1,2,\cdots,n)$满足对$i=1,2,\cdots,n$,都有$\mathop \sum \limits_{k = 1}^i {a_k}≥\sqrt i$。求证:\[\mathop \sum \limits_{i = 1}^n {a_i}^2 \ge \frac{1}{4}H(i)\]
23.设实数$a_i(i=1,2,\cdots,n)\in[0,1]$之和为1,求证\[\mathop \sum \limits_{i = 1}^n \frac{{{x_i}}}{{1 + x_i^2}} \le \frac{{{n^2}}}{{1 + {n^2}}}\]
24.正数${x_i},{y_i},{z_i},{w_i}$满足${x_i}{y_i}{z_i} > w_i^2(i = 1,2)$,求证\[\sqrt[3]{{({x_1} + {x_2})({y_1} + {y_2})({z_1} + {z_2}) - {{({w_1} + {w_2})}^3}}} \ge \sqrt[3]{{{x_1}{y_1}{z_1} - {w_1}^2}} + \sqrt[3]{{{x_2}{y_2}{z_2} - {w_2}^2}}\]
25.${a_i},{b_i} \in {{\bf{R}}^ + },i = 1,2, \cdots ,n(n \ge 2).$求证:\[\left( {\sum\limits_{i = 1}^n {{a_i}^2} } \right)\left( {\sum\limits_{i = 1}^n {{b_i}^2} } \right) \ge \left( {\sum\limits_{i = 1}^n {{b_i}\left( {{a_i} + {b_i}} \right)} } \right)\left( {\sum\limits_{i = 1}^n {\frac{{a_i^2{b_i}}}{{{a_i} + {b_i}}}} } \right)\]
26.${a_1},{a_2},{a_3} \in \left( {0,1} \right),$求证\[\mathop \sum \limits_{k = 1}^3 \left( {\frac{1}{{1 - a_k^2{a_{k + 1}}}} + \frac{1}{{1 - {a_k}a_{k + 1}^2}} - \frac{1}{{1 - a_k^3}}} \right) \le \frac{3}{{1 - {a_1}{a_2}{a_3}}}\]
27.设f(x)是[0,1]上的有界正值函数,证明\[\exists {x_1},{x_2} \in [0,1],s.t.\frac{{\left( {{x_2} - {x_1}} \right){f^2}\left( {{x_1}} \right)}}{{f\left( {{x_2}} \right)}} > \frac{{f(0)}}{4}\]
28.$x,y \in {\bf{C}},|x| = |y| = 1,$求\[F = \left| {1 + x} \right| + \left| {1 + xy} \right| + \left| {1 + x{y^2}} \right| +  \cdots  + \left| {1 + x{y^{2011}}} \right| - 1006\left| {1 + y} \right|\]的最大值
29.$n,k\in\mathbb N,$证明:有k个偶数的n的整数分拆的数目等于最大的重复的数为k的n的整数分拆的数目
有k个偶数的n的整数分拆的数目:取共轭的Ferrers图
奇数出现任意次
2k出现不少于1次$\prod\limits_{i = 1}^{k - 1} {\frac{1}{{1 - {2^i}}}}$
>2k出现0次$\frac{{{x^{2k}}}}{{1 - {x^{2k}}}}$
2,4,$\cdots$,2k-2出现任意次$\prod\limits_{i = 1}^\infty  {\frac{1}{{1 - {x^{2i - 1}}}}} $

最大的重复的数为k的n的整数分拆的数目:
>k的数出现0,1次,$\prod\limits_{i = k + 1}^\infty  {\left( {1 + {x^i}} \right)} $
k出现不少于2次,$\frac{{{x^{2k}}}}{{1 - {x^k}}}$
<k的数出现任意次,$\prod\limits_{i = 1}^{k - 1} {\frac{1}{{1 - {x^i}}}} $
现在要证明:\[\left( {\prod\limits_{i = 1}^{k - 1} {\frac{1}{{1 - {2^i}}}} } \right)\frac{{{x^{2k}}}}{{1 - {x^{2k}}}}\left( {\prod\limits_{i = 1}^\infty  {\frac{1}{{1 - {x^{2i - 1}}}}} } \right) = \left( {\prod\limits_{i = k + 1}^\infty  {\left( {1 + {x^i}} \right)} } \right)\frac{{{x^{2k}}}}{{1 - {x^k}}}\left( {\prod\limits_{i = 1}^{k - 1} {\frac{1}{{1 - {x^i}}}} } \right)\]
30.$U = \{ 1,2, \cdots ,n\} ,i \in U,{A_i} \cap {B_i} = {B_i} \cap {C_i} = {C_i} \cap {A_i} = \emptyset ,{A_i} \cup {B_i} \cup {C_i} = U - \{ i\} ,|{A_i}| = |{B_i}|,\forall i,j \in U,|{A_i} \cap {B_j}| + |{B_i} \cap {C_j}| + |{C_i} \cap {A_j}| = |{A_j} \cap {B_i}| + |{B_j} \cap {C_i}| + |{C_j} \cap {A_i}|$,证明\[i \in {A_j} \Leftrightarrow j \in {A_j},i \in {C_j} \Leftrightarrow j \in {C_i}\]
有人说类似stone-scissors-cloth定理,如果您了解它的出题背景,能否解释一下,如果有出处就更好了
将${C_i} = U -\{ i \}\cup {A_i} \cup {B_i}$代入条件得\[|{A_i} \cap {B_j}| + |{B_i}| - |{B_i} \cap \{ j\} | - |{B_i} \cap {A_j}| - |{B_i} \cap {B_j}| + |{A_j}| - |{A_j} \cap \{ i\} | - |{A_i} \cap {A_j}| - |{A_j} \cap {B_i}| = |{A_j} \cap {B_i}| + |{B_j}| - |{B_j} \cap \{ i\} | - |{B_j} \cap {A_i}| - |{B_j} \cap {B_i}| + |{A_i}| - |\{ j\}  \cap {A_i}| - |{A_j} \cap {A_i}| - |{A_i} \cap {B_j}|\]化简得\[|{A_i} \cap {B_j}| = |{A_j} \cap {B_i}|\],由容斥原理得\[|{A_i}| + |{B_j}| - |{A_i} \cup {B_j}| = |{A_j}| + |{B_i}| - |{A_j} \cup {B_i}|\],即\[|{A_i} \cup {B_j}| = |{A_j} \cup {B_i}|\]
31.闵可夫斯基定理:坐标平面上任何包含原点的、面积大于4的、凸的、关于原点对称的闭区域一定含有异于原点的整点
证明:任取一个关于原点对称且面积大于4的封闭凸图形,一定存在两点,使横纵坐标之差为整数。
设其中一点坐标为$(x_0,y_0)$,另一点为$(x_0+k,y_0+b)(k,b∈\mathbf Z)$,并且$(x_0,y_0)$、$(x_0+k,y_0+b)$都在图形内。因图形关于原点对称,所以对于任意点(x,y),若其在图形中,则关于原点的对称点(-x,-y)也在图形中。所以$(-x_0-k,-y_0-b)$在图形中。连接点$(x_0,y_0)$和点$(-x_0-k,-y_0-b)$,取中点$\left(\frac{x_0+(-x_0-k)}2,\frac{y_0+-y_0-b}2\right)$,由图形为凸区域知,中点在图形内。将图形以原点为位似中心,扩大两倍。中点则为(k,b),新图形面积大于4,且中点是整点,位于图形内。
对于任意一个满足条件的图形,都可以先缩小,找到中点后扩大,这样一定有一异于原点的整点在图形内,命题得证。
闵可夫斯基定理是卡拉西奥多里定理对于紧凸集的精确化。
在有些文献中,也把凸集分离定理称为闵可夫斯基定理。
我对这个证明(来自百度百科)有疑问:为什么一开始就可以取出这样两个点,好像没有说明它们的存在性

TOP

返回列表 回复 发帖